Tabela gruczołów i ich funkcji. W zależności od cech genetycznych i pochodzenia

Gruczoły dokrewne lub gruczoły dokrewne (ZHVS) nazywane są narządami gruczołowymi, których sekret wchodzi bezpośrednio do krwi. W przeciwieństwie do gruczołów wydzielania zewnętrznego, których produkty wchodzą do jam ciała komunikujących się ze środowiskiem zewnętrznym, przewód pokarmowy nie ma przewodów wydalniczych. Ich sekrety nazywane są hormonami. Uwolnione do krwi są przenoszone przez całe ciało i oddziałują na różne układy narządów.

Jakie są gruczoły dokrewne

W tabeli przedstawiono narządy związane z gruczołami dokrewnymi oraz wytwarzane przez nie hormony:

*Trzustka posiada zarówno wydzielinę zewnętrzną, jak i wewnętrzną.

W niektórych źródłach grasica (gruczoł grasicy) określana jest również jako gruczoły dokrewne, w których powstają substancje niezbędne do regulacji funkcjonowania układu odpornościowego. Jak wszystkie VVS, tak naprawdę nie ma kanalików i wydziela swoje produkty bezpośrednio do krwiobiegu. Jednak grasica aktywnie działa do wieku dojrzewania, wtedy następuje inwolucja (zastąpienie miąższu tkanką tłuszczową).

Anatomia i funkcje aparatu dokrewnego

Wszystkie gruczoły dokrewne mają różną anatomię i zestaw syntetyzowanych hormonów, dlatego funkcje każdego z nich są radykalnie różne.

Należą do nich podwzgórze, przysadka, nasada, tarczyca, przytarczyce, trzustka i gonady, nadnercza.

Podwzgórze

Podwzgórze jest ważną anatomiczną formacją ośrodkowego układu nerwowego, która ma silne ukrwienie i jest dobrze unerwiona. Oprócz regulacji wszystkich autonomicznych funkcji organizmu wydziela hormony stymulujące lub hamujące pracę przysadki mózgowej (hormony uwalniające).

Środki aktywujące:

  • tyroliberyna;
  • kortykoliberyna;
  • gonadoliberyna;
  • somatoliberyna.

Do hormonów podwzgórza hamujących czynność przysadki należą:

  • somatostatyna;
  • melanostatyna.

Większość czynników uwalniających podwzgórza nie jest selektywna. Każdy działa natychmiast na kilka hormonów tropikalnych przysadki mózgowej. Na przykład tyroliberyna aktywuje syntezę tyreotropiny i prolaktyny, a somatostatyna hamuje tworzenie większości hormonów peptydowych, ale głównie hormonu wzrostu i kortykotropiny.

W przednio-bocznym obszarze podwzgórza znajdują się skupiska specjalnych komórek (jąder), w których tworzą się wazopresyna (hormon antydiuretyczny) i oksytocyna.

Wazopresyna, działając na receptory dystalnych kanalików nerkowych, stymuluje zwrotne wchłanianie zwrotne wody z moczu pierwotnego, tym samym zatrzymując płyn w organizmie i zmniejszając diurezę. Innym efektem działania substancji jest wzrost całkowitego obwodowego oporu naczyniowego (skurczu naczyń) oraz wzrost ciśnienia krwi.

Oksytocyna ma w niewielkim stopniu takie same właściwości jak wazopresyna, ale jej główną funkcją jest stymulacja porodu (skurczy macicy), a także zwiększenie wydzielania mleka z gruczołów sutkowych. Zadanie tego hormonu w męskim ciele nie zostało jeszcze ustalone.

Przysadka

Przysadka mózgowa jest centralnym gruczołem w ludzkim organizmie, który reguluje pracę wszystkich gruczołów przysadkowo zależnych (z wyjątkiem trzustki, szyszynki i przytarczyc). Znajduje się w tureckim siodle kości klinowej, ma bardzo małe wymiary (waga około 0,5 g; średnica - 1 cm). Jest podzielony na 2 płaty: przedni (adenohypophysis) i tylny (neurohypophysis). Szypułka przysadki, która jest połączona z podwzgórzem, dostarcza hormony uwalniające do przysadki mózgowej, a oksytocynę i wazopresynę do przysadki mózgowej (gdzie się kumulują).

Przysadka w tureckim siodle kości klinowej. Przysadka mózgowa jest pomalowana na jasnoróżowo, neuroprzysadka jest pomalowana na jasnoróżowo.

Hormony, za pomocą których przysadka kontroluje gruczoły obwodowe, nazywane są tropikami. Regulacja powstawania tych substancji następuje nie tylko dzięki czynnikom uwalniającym podwzgórza, ale także produktom aktywności samych gruczołów obwodowych. W fizjologii mechanizm ten nazywa się negatywnym sprzężeniem zwrotnym. Na przykład przy nadmiernie wysokiej produkcji hormonów tarczycy hamowana jest synteza tyreotropiny, a wraz ze spadkiem poziomu hormonów tarczycy wzrasta jej stężenie.

Prolaktyna jest jedynym nietropowym hormonem przysadki mózgowej (czyli nie realizuje swojego działania kosztem innych gruczołów). Jego głównym zadaniem jest stymulacja laktacji u kobiet karmiących.

Hormon somatotropowy (somatotropina, hormon wzrostu, hormon wzrostu) również warunkowo odnosi się do tropiku. Główną rolą tego peptydu w organizmie jest stymulowanie rozwoju. Jednak efekt ten nie jest realizowany przez samo STG. Aktywuje powstawanie w wątrobie tzw. insulinopodobnych czynników wzrostu (somatomedyn), które mają stymulujący wpływ na rozwój i podział komórek. STH wywołuje szereg innych efektów, np. bierze udział w metabolizmie węglowodanów poprzez aktywację glukoneogenezy.

Hormon adrenokortykotropowy (kortykotropina) to substancja regulująca pracę kory nadnerczy. Jednak ACTH prawie nie ma wpływu na tworzenie aldosteronu. Jego synteza jest regulowana przez układ renina-angiotensyna-aldosteron. Pod wpływem ACTH aktywowana jest produkcja kortyzolu i sterydów płciowych w nadnerczach.

Hormon tyreotropowy (tyrotropina) działa stymulująco na czynność tarczycy, zwiększając tworzenie tyroksyny i trójjodotyroniny.

Hormony gonadotropowe - stymulujące pęcherzyki (FSH) i luteinizujące (LH) aktywują aktywność gonad. U mężczyzn są niezbędne do regulacji syntezy testosteronu i tworzenia plemników w jądrach, u kobiet do realizacji owulacji i tworzenia estrogenów i progestagenów w jajnikach.

Epifiza

Szyszynka jest małym gruczołem ważącym zaledwie 250 mg. Ten narząd dokrewny znajduje się w okolicy śródmózgowia.

Funkcja szyszynki nie została jeszcze w pełni poznana. Jedynym znanym związkiem jest melatonina. Ta substancja to „zegar wewnętrzny”. Zmieniając jego koncentrację, organizm człowieka rozpoznaje porę dnia. To właśnie z funkcją nasady wiąże się adaptacja do innych stref czasowych.

Tarczyca

Gruczoł tarczycy (TG) znajduje się na przedniej powierzchni szyi pod chrząstką tarczycy krtani. Składa się z 2 płatów (prawy i lewy) oraz przesmyku. W niektórych przypadkach dodatkowy płat piramidalny odchodzi od przesmyku.

Wielkość tarczycy jest bardzo zmienna, dlatego przy określaniu zgodności z normą mówią o objętości tarczycy. U kobiet nie powinna przekraczać 18 ml, u mężczyzn - 25 ml.

W tarczycy powstają tyroksyna (T4) i trójjodotyronina (T3), które odgrywają ważną rolę w życiu człowieka, wpływając na procesy metaboliczne wszystkich tkanek i narządów. Zwiększają zużycie tlenu przez komórki, stymulując tym samym produkcję energii. Przy ich niedoborze organizm cierpi na głód energetyczny, a przy nadmiarze rozwijają się procesy dystroficzne w tkankach i narządach.

Hormony te są szczególnie ważne w okresie wzrostu wewnątrzmacicznego, ponieważ ich niedobór zakłóca powstawanie mózgu płodu, czemu towarzyszy upośledzenie umysłowe i upośledzony rozwój fizyczny.

Kalcytonina jest produkowana w komórkach C tarczycy, których główną funkcją jest obniżanie poziomu wapnia we krwi.

przytarczyce

Gruczoły przytarczyczne znajdują się na tylnej powierzchni tarczycy (w niektórych przypadkach wchodzą w skład tarczycy lub znajdują się w nietypowych miejscach - grasicy, rowku przytchawiczym itp.). Średnica tych zaokrąglonych formacji nie przekracza 5 mm, a liczba może wynosić od 2 do 12 par.

Schematyczne ułożenie przytarczyc.

Gruczoły przytarczyczne wytwarzają hormon przytarczyc, który wpływa na metabolizm fosforowo-wapniowy:

  • zwiększa resorpcję kości, uwalniając z kości wapń i fosfor;
  • zwiększa wydalanie fosforu z moczem;
  • stymuluje tworzenie kalcytriolu w nerkach (aktywnej postaci witaminy D), co prowadzi do zwiększonego wchłaniania wapnia w jelicie.

Pod wpływem parathormonu następuje wzrost poziomu wapnia i spadek stężenia fosforu we krwi.

nadnercza

Prawe i lewe nadnercza znajdują się nad górnymi biegunami odpowiednich nerek. Prawa przypomina zarys trójkąta, a lewa półksiężyca. Te gruczoły ważą około 20 g.

Przekrój nadnerczy (schemat). Substancja korowa jest podświetlona światłem, rdzeń jest podświetlony na ciemno.

Na przecięciu nadnercza izoluje się substancje korowe i rdzeniowe. Pierwsza zawiera 3 mikroskopijne warstwy funkcjonalne:

  • kłębuszkowy (synteza aldosteronu);
  • wiązka (produkcja kortyzolu);
  • siatkowa (synteza sterydów płciowych).

Aldosteron odpowiada za regulację równowagi elektrolitowej. Pod jego działaniem w nerkach wzrasta reabsorpcja zwrotna sodu (i wody) oraz wydalanie potasu.

Kortyzol ma różnorodny wpływ na organizm. To hormon, który przystosowuje człowieka do stresu. Główne funkcje:

  • wzrost poziomu glukozy we krwi w wyniku aktywacji glukoneogenezy;
  • zwiększony rozkład białek;
  • specyficzny wpływ na metabolizm tłuszczów (zwiększona synteza lipidów w podskórnej tkance tłuszczowej górnej części ciała oraz zwiększony rozkład w tkance kończyn);
  • zmniejszona reaktywność układu odpornościowego;
  • hamowanie syntezy kolagenu.

Sterydy płciowe (androstendion i dihydroepiandrosteron) wywołują efekty podobne do testosteronu, ale są od niego gorsze pod względem działania androgennego.

W rdzeniu nadnerczy syntetyzowana jest adrenalina i noradrenalina, które są hormonami układu współczulnego-nadnerczowego. Ich główne efekty:

  • zwiększona częstość akcji serca, zwiększona pojemność minutowa serca i ciśnienie krwi;
  • skurcz wszystkich zwieraczy (zatrzymanie moczu i defekacja);
  • spowolnienie wydzielania gruczołów zewnątrzwydzielniczych;
  • wzrost światła oskrzeli;
  • rozszerzenie źrenic;
  • podwyższony poziom glukozy we krwi (aktywacja glukoneogenezy i glikogenolizy);
  • przyspieszenie metabolizmu w tkance mięśniowej (glikoliza tlenowa i beztlenowa).

Działanie tych hormonów ma na celu szybką aktywację organizmu w warunkach awaryjnych (konieczność lotu, ochrona itp.).

Aparat dokrewny trzustki

Zgodnie ze swoim znaczeniem trzustka jest narządem o mieszanej wydzielinie. Ma układ przewodowy, przez który enzymy trawienne dostają się do jelit, ale zawiera również układ hormonalny - wysepki Langerhansa, z których większość znajduje się w ogonie. Wytwarzają następujące hormony:

  • insulina (komórki beta wysepek);
  • glukagon (komórki alfa);
  • somatostatyna (komórki D).

Insulina reguluje różne rodzaje metabolizmu:

  • obniża poziom glukozy we krwi poprzez stymulację wnikania glukozy do tkanek insulinozależnych (tkanki tłuszczowej, wątroby i mięśni), hamuje procesy glukoneogenezy (synteza glukozy) i glikogenolizy (rozpad glikogenu);
  • aktywuje produkcję białka i tłuszczów.

Glukagon jest hormonem przeciwstawnym. Jego główną funkcją jest aktywacja glikogenolizy.

Somatostatyna hamuje produkcję insuliny i glukagonu.

gonady

Gonady produkują sterydy płciowe.

U mężczyzn testosteron jest głównym hormonem płciowym. Jest produkowany w jądrach (komórkach Leydiga), które zwykle znajdują się w mosznie i mają średni rozmiar 35-55 i 20-30 mm.

Główne funkcje testosteronu:

  • stymulacja wzrostu szkieletu i rozmieszczenie tkanki mięśniowej w zależności od typu męskiego;
  • rozwój genitaliów, strun głosowych, pojawienie się męskiego owłosienia na ciele;
  • tworzenie męskiego stereotypu zachowań seksualnych;
  • udział w spermatogenezie.

W przypadku kobiet głównymi sterydami płciowymi są estradiol i progesteron. Hormony te są wytwarzane w pęcherzykach jajnikowych. W dojrzewającym pęcherzyku główną substancją jest estradiol. Po pęknięciu pęcherzyka w czasie owulacji na jego miejscu tworzy się ciałko żółte, które wydziela głównie progesteron.

Jajniki u kobiet znajdują się w miednicy małej po bokach macicy i mierzą 25-55 i 15-30 mm.

Główne funkcje estradiolu:

  • budowa ciała, rozkład tłuszczu podskórnego w zależności od typu żeńskiego;
  • stymulacja proliferacji nabłonka przewodowego gruczołów sutkowych;
  • aktywacja tworzenia warstwy funkcjonalnej endometrium;
  • stymulacja szczytu owulacyjnego hormonów gonadotropowych;
  • tworzenie kobiecego typu zachowań seksualnych;
  • stymulacja pozytywnego metabolizmu kostnego.

Główne efekty progesteronu:

  • stymulacja czynności wydzielniczej endometrium i jej przygotowanie do implantacji zarodka;
  • tłumienie czynności skurczowej macicy (zachowanie ciąży);
  • stymulacja różnicowania nabłonka przewodowego gruczołów sutkowych, przygotowanie ich do laktacji.

A ich hormony (zwane również sekretami) zapewniają funkcjonowanie układu hormonalnego organizmu. Sekrety są wydzielane do wewnętrznego środowiska ciała, ponieważ te narządy nie mają kanałów, które umożliwiają usuwanie wydzieliny do jamy lub na powierzchnię skóry.

Narządy wydzielające substancje biologicznie czynne dzielą się na trzy duże grupy: wydzielinę zewnętrzną, wewnętrzną i mieszaną.

  • Narządy wydzielania zewnętrznego to gruczoły potowe, łojowe, ślinowe i żołądkowe. Uwolniony sekret przechodzi przewodami na powierzchnię skóry, jamy ustnej lub żołądka.
  • Grupa narządów dokrewnych wydzielania wewnętrznego obejmuje przysadkę mózgową, nadnercza, tarczycę i przytarczyce. Krew jest głównym nośnikiem tych tajemnic. Przychodzą tu hormony wydzielane przez gruczoły z wydzieliną wewnętrzną.
  • Grasica, trzustka i gonady są klasyfikowane jako wydzieliny mieszane. Dotyczy to również łożyska. Tradycyjnie nazywa się je układem hormonalnym, ponieważ hormon może być uwalniany zarówno na zewnątrz, jak i wewnątrz organizmu.

Główną funkcją układu hormonalnego jest regulacja procesów zachodzących w organizmie. Dojrzewanie komórki jajowej lub nasienia, początek dojrzewania lub menopauzy, depresja, bezsenność i nadmierna aktywność – konsekwencje działania substancji mogą być różne, a ich działanie złożone i zrównoważone.

Anatomicznie ten obszar mózgu nie jest narządem wydzielniczym, ponieważ jest reprezentowany przez neurony. Ale ten ostatni może wydzielać substancje aktywujące pracę przysadki mózgowej - kolejnego przedstawiciela narządów wydzielania wewnętrznego.

W ten sposób przedstawiona jest praca. Hormony są syntetyzowane w neuronach i wytwarzane w przysadce mózgowej, skąd przedostają się do krwiobiegu i docierają do narządu docelowego. Głównymi sekretami gruczołu i hormonów wytwarzanych w wyniku ich działania są wazopresyna.

  • Prolaktyna odpowiada za początek okresu laktacji i tworzenie mleka u kobiet w ciąży.
  • Oksytocyna stymuluje pracę mięśni gładkich, wzmacnia mięśnie i aktywność skurczową włókien mięśniowych. Wskazany dla kobiet w ciąży z niską aktywnością włókien mięśniowych macicy, a także z hipotrofią mięśni.
  • Wazopresyna reguluje wydalanie wody przez nerki, zwiększa napięcie mięśni gładkich przewodu pokarmowego, a przy nadmiarze wydzieliny podnosi ciśnienie krwi.

Przysadka

Wierzchołkiem gruczołów dokrewnych jest przysadka mózgowa. Znajduje się w centrum mózgu, a jego wymiary nie przekraczają 5x5 mm. Jest kilka celów, w które wchodzą. Reguluje pracę innych gruczołów, układu rozrodczego, procesy metaboliczne i wzrost człowieka.

Przysadka wydziela kortykotropinę, wydzielinę tyreotropową i gonadotropową.

  • Kortykotropina reguluje pracę nadnerczy, stymuluje wydzielanie w nich hormonów
  • Tyrotropina stymuluje produkcję: tyroksyny i trójjodotyroniny, które dodatkowo regulują procesy metaboliczne i stan skóry
  • Folitropina odpowiada za powstawanie pęcherzyków, a lutropina za pękanie błony pęcherzyka i tworzenie ciałka żółtego.
  • Somatotropina jest najważniejszym hormonem tworzonym przez gruczoł dokrewny. Uwalniany do krwi i jam ciała zwiększa syntezę RNA, reguluje metabolizm węglowodanów i stymuluje procesy wzrostu. Brak somatotropiny w dzieciństwie prowadzi do życia.

Tarczyca

Narząd w postaci tarczy znajduje się na przedniej ścianie szyi i osiąga masę 20-23 g. Pod działaniem przysadki aktywuje się synteza sekretów w komórkach A tarczycy , po czym są uwalniane do krwi, gdzie wiążą się z białkami nośnikowymi i docierają do narządów docelowych.

Tarczyca i przytarczyce wydzielają tyroksynę, kalcytoninę i trójjodotyroninę. Pierwsze dwa hormony są określane skrótem T4 i T3.

  • - hormonalny regulator metabolizmu i syntezy peptydów. Uczestniczy w rozwoju i wzroście organizmu. Nadmiar T4 to powszechna choroba endokrynologiczna, gdy wytwarzany hormon jest odrzucany przez organizm i jest przez niego traktowany jako substancja obca.
  • Trijodotyronina, której produkcja tylko jedna czwarta zachodzi w tarczycy, bierze również udział w regulacji procesów metabolicznych i syntezie białek, uwalnianych z T4.
  • bierze czynny udział we wzmacnianiu tkanki kostnej, zmniejsza stężenie fosforu i wapnia we krwi, aktywuje wydalanie fosforanów przez nerki.

trzustka

Gruczoły mieszane wytwarzają hormony o funkcji zarówno wewnątrz-, jak i zewnątrzwydzielniczej. Ostatnią funkcję pełnią małe wysepki trzustkowe, które przebijają naczynia włosowate.

Hormony utworzone przez wysepki dostają się do tych naczyń włosowatych przez błony śródbłonka i są przenoszone przez krew w całym ciele.

  • - wydzielanie hormonu zachodzi w komórkach A wysepek. Jego funkcja ma na celu przekształcenie przychodzącego glikogenu w bardziej przyswajalną formę - glukozę.
  • - najważniejszy hormon odpowiedzialny za regulację poziomu glukozy we krwi. Za każdym razem, gdy glukoza dostaje się do krwiobiegu, insulina wiąże ją w skrobię zwierzęcą, która jest spalana przez włókna mięśniowe. Spadek wydzielania insuliny prowadzi do cukrzycy, a wzrost do nadmiernego spożycia glukozy przez tkanki, odkładania się cukrów i śpiączki hipoglikemicznej.
  • Polipeptyd trzustkowy i somatostatyna to substancje o ogólnym podłożu hormonalnym, które mają niewielkie znaczenie w praktyce klinicznej.

nadnercza

Jest to sparowany narząd dokrewny, który tworzy sygnalizacyjne układy hormonalne organizmu. Znajduje się nad górną częścią nerek i osiąga masę nie większą niż 8 g. Wydzielanie następuje w korze narządu.

Rozwój i funkcjonowanie kory jest całkowicie zależne od przysadki mózgowej.

  • - substancja sygnalizacyjna, która przyspiesza bicie serca, obkurcza naczynia krwionośne i przyspiesza syntezę glukozy. Wzrasta pobudliwość siatkówki, aparatów przedsionkowych i słuchowych – organizm pracuje w trybie „awaryjnym” pod wpływem bodźców zewnętrznych.
  • - zwiastun adrenaliny. Syntetyzowana jest przed adrenaliną, a w przypadku ekstremalnych bodźców natychmiast przekształcana jest w ostateczną formę.
  • - reguluje metabolizm soli, zapobiegając hiperkaliemii.

Należą do nich jądra i jajniki. Wiedząc, gdzie idą hormony wydzielane przez gruczoły dokrewne, łatwo jest zrozumieć zasadę działania gruczołów płciowych.

Jądra wytwarzają męskie hormony płciowe (androgeny), które wpływają na rozwój i funkcjonowanie układu rozrodczego.

Jajniki produkują - żeńskie hormony płciowe, które są odpowiedzialne za początek ciąży, funkcje rozrodcze, a także stymulują produkcję mleka matki.

Wniosek

Nie da się powiedzieć, które gruczoły są ważniejsze dla organizmu, ponieważ ich system pracy jest ze sobą powiązany i zależny od każdego hormonu. Hormony tworzone przez gruczoły dokrewne są stale wydzielane, zapewniając niezbędne funkcje organizmu.

Naruszenia w pracy jednego narządu dokrewnego pociągną za sobą zmiany nie tylko w innych gruczołach, ale we wszystkich narządach. Z tego powodu większość diagnoz rozpoczyna się od analizy układu hormonalnego w celu określenia, które hormony znajdują się poza normalnym zakresem.

Bibliografia

  1. Grebenshchikov Yu.B., Moshkovsky Yu.Sh., Chemia bioorganiczna // Właściwości fizyczne i chemiczne, struktura i aktywność funkcjonalna insuliny. - 1986. - s.296.
  2. Filippovich Yu.B., Podstawy biochemii // Hormony i ich rola w metabolizmie. - 1999r. - s. 451-453, 455-456, 461-462.
  3. Fizjologia człowieka / wyd. GI Kositsky. - 3. ed., poprawione. i dodatkowe - M.: Medycyna, 1985, 544 s.;
  4. Tepperman J., Tepperman H., Fizjologia metabolizmu i układu hormonalnego. Kurs wprowadzający. - Za. z angielskiego. - M.: Mir, 1989. - 656 s.; Fizjologia. Podstawy i układy funkcjonalne: tok wykładów / wyd. K. V. Sudakowa. – M.: Medycyna. - 2000. -784 s.;
  5. Agadzhanyan M. A., Smirnov V. M., Normalna fizjologia: podręcznik dla studentów medycyny. - M .: Wydawnictwo LLC "Agencja Informacji Medycznej", - 2009. - 520 s.;
  6. Anosova L. N., Zefirova G. S., Krakov V. A. Krótka endokrynologia. – M.: Medycyna, 1971.

1. Fizjologiczna rola gruczołów dokrewnych. Charakterystyka działania hormonów.

Gruczoły dokrewne to wyspecjalizowane narządy, które mają budowę gruczołową i wydzielają swój sekret do krwi. Nie mają przewodów wydalniczych. Do gruczołów tych należą: przysadka, tarczyca, przytarczyce, nadnercza, jajniki, jądra, grasica, trzustka, szyszynka, system APUD (system wychwytywania prekursorów amin i ich dekarboksylacji), a także serce - wytwarza przedsionki sód - czynnik moczopędny, nerki - wytwarzają erytropoetynę, reninę, kalcytriol, wątroba - wytwarzają somatomedynę, skóra - wytwarza kalcyferol (witamina D 3), przewód pokarmowy - wytwarza gastrynę, sekretynę, cholecystokininę, VIP (peptyd wazo-jelitowy), GIP (żołądkowy peptyd hamujący ).

Hormony pełnią następujące funkcje:

Uczestniczą w utrzymaniu homeostazy środowiska wewnętrznego, kontrolują poziom glukozy, objętość płynu pozakomórkowego, ciśnienie krwi, równowagę elektrolitową.

Zapewnij rozwój fizyczny, seksualny, umysłowy. Odpowiadają również za cykl rozrodczy (cykl menstruacyjny, owulacja, spermatogeneza, ciąża, laktacja).

Kontroluj tworzenie i wykorzystanie składników odżywczych i zasobów energetycznych w organizmie

Hormony zapewniają procesy adaptacji układów fizjologicznych do działania bodźców środowiska zewnętrznego i wewnętrznego oraz uczestniczą w reakcjach behawioralnych (zapotrzebowanie na wodę, pokarm, zachowania seksualne)

Są mediatorami w regulacji funkcji.

Gruczoły dokrewne tworzą jeden z dwóch systemów regulujących funkcje. Hormony różnią się od neuroprzekaźników tym, że zmieniają reakcje chemiczne w komórkach, na które działają. Mediatory wywołują reakcję elektryczną.

Termin „hormon” pochodzi od greckiego słowa HORMAE – „ekscytuję, zachęcam”.

Klasyfikacja hormonów.

Według struktury chemicznej:

1. Hormony steroidowe – pochodne cholesterolu (hormony kory nadnerczy, gonady).

2. Hormony polipeptydowe i białkowe (przedni przysadka mózgowa, insulina).

3. Pochodne aminokwasu tyrozyny (adrenalina, noradrenalina, tyroksyna, trijodotyronina).

Funkcjonalnie:

1. Hormony tropowe (aktywują aktywność innych gruczołów dokrewnych; są to hormony przedniego płata przysadki mózgowej)

2. Hormony efektorowe (działają bezpośrednio na procesy metaboliczne w komórkach docelowych)

3. Neurohormony (uwalniane w podwzgórzu - liberyny (aktywujące) i statyny (hamujące)).

właściwości hormonów.

zdalny charakter działania (np. hormony przysadki wpływają na nadnercza),

Ścisła specyfika hormonów (brak hormonów prowadzi do utraty określonej funkcji, a temu procesowi można zapobiec tylko poprzez wprowadzenie niezbędnego hormonu),

Posiadają wysoką aktywność biologiczną (powstają w niskich stężeniach w kwasie tłuszczowym).

Hormony nie mają zwyczajnej specyfiki,

Mają krótki okres półtrwania (szybko niszczone przez tkanki, ale mają długi efekt hormonalny).

2. Mechanizmy hormonalnej regulacji funkcji fizjologicznych. Jego cechy w porównaniu z regulacją nerwową. Systemy powiązań bezpośrednich i odwrotnych (dodatnich i ujemnych). Metody badania układu hormonalnego.

Wydzielanie wewnętrzne (inkrecja) to uwalnianie wyspecjalizowanych substancji biologicznie czynnych - hormony- do wewnętrznego środowiska organizmu (krew lub limfa). Termin "hormon" został po raz pierwszy zastosowany do sekretyny (hormonu jelita 12) przez Starlinga i Beilisa w 1902 roku. Hormony różnią się od innych substancji biologicznie czynnych, na przykład metabolitów i mediatorów, tym, że po pierwsze są tworzone przez wysoce wyspecjalizowane komórki endokrynologiczne, a po drugie wpływają na tkanki odległe od gruczołu poprzez środowisko wewnętrzne, tj. mają odległy efekt.

Najstarszą formą regulacji jest: humoralno-metaboliczny(dyfuzja substancji czynnych do sąsiednich komórek). Występuje w różnych postaciach u wszystkich zwierząt, szczególnie wyraźnie objawia się w okresie embrionalnym. Rozwijając się układ nerwowy podporządkował sobie regulację humoralno-metaboliczną.

Prawdziwe gruczoły dokrewne pojawiły się późno, ale we wczesnych stadiach ewolucji są neurosekrecja. Neurosekrety nie są neuroprzekaźnikami. Mediatory są związkami prostszymi, działają lokalnie w okolicy synapsy i szybko ulegają zniszczeniu, natomiast neurosekrecje to substancje białkowe, które wolniej się rozkładają i działają na duże odległości.

Wraz z pojawieniem się układu krążenia do jego jamy zaczęły uwalniać się neurosekrecje. Następnie powstały specjalne formacje do gromadzenia i zmiany tych sekretów (w pierścienicach), następnie ich wygląd stał się bardziej skomplikowany, a same komórki nabłonkowe zaczęły wydzielać swoje sekrety do krwi.

Narządy dokrewne mają bardzo odmienne pochodzenie. Część z nich powstała z narządów zmysłu (szyszynka – z trzeciego oka), inne gruczoły dokrewne powstały z gruczołów wydzielania zewnętrznego (tarczycy). Z pozostałości organów tymczasowych (grasicy, przytarczyc) powstały gruczoły rozgałęzione. Gruczoły steroidowe powstały z mezodermy, ze ścian celomu. Hormony płciowe są wydzielane przez ściany gruczołów zawierających komórki płciowe. w ten sposób różne narządy dokrewne mają różne pochodzenie, ale wszystkie powstały jako dodatkowy sposób regulacji. Istnieje jedna regulacja neurohumoralna, w której wiodącą rolę odgrywa układ nerwowy.

Dlaczego powstał taki dodatek do regulacji nerwowej? Komunikacja neuronowa - szybka, dokładna, adresowana lokalnie. Hormony – działają szerzej, wolniej, dłużej. Zapewniają długotrwałą reakcję bez udziału układu nerwowego, bez ciągłej impulsacji, co jest nieekonomiczne. Hormony mają długotrwały efekt. Gdy wymagana jest szybka reakcja, działa układ nerwowy. Gdy wymagana jest wolniejsza i stabilniejsza reakcja na powolne i długotrwałe zmiany w środowisku, działają hormony (wiosna, jesień itp.), zapewniając wszelkie zmiany adaptacyjne w organizmie, aż do zachowań seksualnych. U owadów hormony zapewniają całkowitą metamorfozę.

Układ nerwowy działa na gruczoły w następujący sposób:

1. Poprzez włókna neurosekrecyjne autonomicznego układu nerwowego;

2. Poprzez neurosekrety – powstawanie tzw. czynniki uwalniające lub hamujące;

3. Układ nerwowy może zmieniać wrażliwość tkanek na hormony.

Hormony wpływają również na układ nerwowy. Istnieją receptory, które reagują na ACTH, na estrogen (w macicy), hormony wpływają na GNI (seksualne), aktywność formacji siatkowatej i podwzgórza itp. Hormony wpływają na zachowanie, motywację i odruchy oraz biorą udział w reakcji na stres.

Istnieją odruchy, w których część hormonalna jest zawarta jako ogniwo. Na przykład: przeziębienie - receptor - OUN - podwzgórze - czynnik uwalniający - wydzielanie hormonu tyreotropowego - tyroksyna - wzrost metabolizmu komórkowego - wzrost temperatury ciała.

Metody badania gruczołów dokrewnych.

1. Usunięcie gruczołu - wytępienie.

2. Przeszczep gruczołu, wprowadzenie ekstraktu.

3. Chemiczna blokada funkcji gruczołów.

4. Oznaczanie hormonów w mediach płynnych.

5. Metoda izotopów promieniotwórczych.

3. Mechanizmy oddziaływania hormonów z komórkami. Pojęcie komórek docelowych. Rodzaje odbioru hormonów przez komórki docelowe. Pojęcie receptorów błonowych i cytozolowych.

Hormony peptydowe (białkowe) produkowane są w postaci prohormonów (ich aktywacja następuje podczas hydrolitycznego rozszczepienia), hormony rozpuszczalne w wodzie gromadzą się w komórkach w postaci granulek, rozpuszczalne w tłuszczach (steroidy) są uwalniane w trakcie ich powstawania.

W przypadku hormonów we krwi istnieją białka nośnikowe - są to białka transportowe, które mogą wiązać hormony. W tym przypadku nie zachodzą żadne reakcje chemiczne. Część hormonów można przenieść w postaci rozpuszczonej. Hormony są dostarczane do wszystkich tkanek, ale tylko komórki, które mają receptory dla działania hormonu, reagują na działanie hormonów. Komórki niosące receptory nazywane są komórkami docelowymi. Komórki docelowe dzielą się na: zależne od hormonów i

wrażliwe na hormony.

Różnica między tymi dwiema grupami polega na tym, że komórki zależne od hormonów mogą rozwijać się tylko w obecności tego hormonu. (Tak więc na przykład komórki płciowe mogą rozwijać się tylko w obecności hormonów płciowych), a komórki wrażliwe na hormony mogą rozwijać się bez hormonu, ale są w stanie dostrzec działanie tych hormonów. (Tak więc na przykład komórki układu nerwowego rozwijają się bez wpływu hormonów płciowych, ale dostrzegają ich działanie).

Każda komórka docelowa ma specyficzny receptor dla działania hormonu, a niektóre receptory znajdują się w błonie. Ten receptor jest stereospecyficzny. W innych komórkach receptory znajdują się w cytoplazmie - są to receptory cytozolowe, które reagują z hormonem wchodzącym do komórki.

Dlatego receptory dzielą się na błonowe i cytozolowe. Aby komórka zareagowała na działanie hormonu, konieczne jest tworzenie wtórnych przekaźników działania hormonów. Jest to typowe dla hormonów z odbiorem błonowym.

4. Układy wtórnych mediatorów działania hormonów peptydowych i katecholamin.

Wtórnymi mediatorami działania hormonów są:

1. Cyklaza adenylanowa i cykliczny AMP,

2. Cyklaza guanylanowa i cykliczny GMF,

3. Fosfolipaza C:

diacyloglicerol (DAG),

tri-fosforan inozytolu (IF3),

4. Zjonizowany Ca - kalmodulina

Heterotroficzne białko G-białko.

Białko to tworzy pętle w błonie i ma 7 segmentów. Porównuje się je z serpentynowymi wstążkami. Posiada wystającą (zewnętrzną) i wewnętrzną część. Do zewnętrznej części dołączony jest hormon, a na wewnętrznej powierzchni znajdują się 3 podjednostki - alfa, beta i gamma. W stanie nieaktywnym białko to zawiera difosforan guanozyny. Ale po aktywacji difosforan guanozyny zmienia się w trifosforan guanozyny. Zmiana aktywności białka G prowadzi albo do zmiany przepuszczalności jonowej błony, albo do aktywacji układu enzymatycznego (cyklaza adenylanowa, cyklaza guanylanowa, fosfolipaza C) w komórce. Powoduje to powstanie specyficznych białek, aktywuje się kinaza białkowa (niezbędna do procesów fosforylacji).

Białka G mogą być aktywujące (Gs) i hamujące lub innymi słowy hamujące (Gi).

Zniszczenie cyklicznego AMP następuje pod działaniem enzymu fosfodiesterazy. Cykliczny HMF ma odwrotny skutek. Gdy fosfolipaza C jest aktywowana, powstają substancje, które przyczyniają się do gromadzenia zjonizowanego wapnia wewnątrz komórki. Wapń aktywuje kinazy białkowe, wspomaga skurcze mięśni. Diacyloglicerol sprzyja konwersji fosfolipidów błonowych do kwasu arachidonowego, który jest źródłem powstawania prostaglandyn i leukotrienów.

Kompleks receptorów hormonalnych penetruje jądro i działa na DNA, co zmienia procesy transkrypcji i tworzy się mRNA, który opuszcza jądro i trafia do rybosomów.

Dlatego hormony mogą zapewnić:

1. Kinetyczna lub startowa akcja,

2. Działanie metaboliczne,

3. Działanie morfogenetyczne (różnicowanie tkanek, wzrost, metamorfoza),

4. Działania korygujące (korektywne, adaptacyjne).

Mechanizmy działania hormonów w komórkach:

Zmiany w przepuszczalności błon komórkowych,

Aktywacja lub hamowanie układów enzymatycznych,

Wpływ na informację genetyczną.

Regulacja opiera się na ścisłej interakcji układu hormonalnego i nerwowego. Procesy wzbudzania w układzie nerwowym mogą aktywować lub hamować aktywność gruczołów dokrewnych. (Rozważmy na przykład proces owulacji u królika. Owulacja u królika następuje dopiero po akcie krycia, który stymuluje uwalnianie hormonu gonadotropowego z przysadki mózgowej. Ten ostatni powoduje proces owulacji).

Po przeniesieniu urazu psychicznego może wystąpić tyreotoksykoza. Układ nerwowy kontroluje wydzielanie hormonów przysadki (neurohormonu), a przysadka wpływa na aktywność innych gruczołów.

Istnieją mechanizmy sprzężenia zwrotnego. Nagromadzenie hormonu w organizmie prowadzi do zahamowania produkcji tego hormonu przez odpowiedni gruczoł, a niedobór będzie mechanizmem stymulującym powstawanie hormonu.

Istnieje mechanizm samoregulacji. (Na przykład, glukoza we krwi determinuje produkcję insuliny i/lub glukagonu; jeśli poziom cukru wzrasta, wytwarzana jest insulina, a jeśli spada, wytwarzany jest glukagon. Brak Na stymuluje produkcję aldosteronu.)

6. Przysadka mózgowa, jej związek z podwzgórzem. Charakter działania hormonów przedniego płata przysadki mózgowej. Hipo- i nadmierne wydzielanie hormonów przysadki mózgowej. Związane z wiekiem zmiany w tworzeniu hormonów przedniego płata.

Komórki przysadki mózgowej (zobacz ich budowę i skład w badaniu histologicznym) wytwarzają hormony: somatotropinę (hormon wzrostu), prolaktynę, tyreotropinę (hormon tyreotropowy), hormon folikulotropowy, hormon luteinizujący, kortykotropinę (ACTH), melanotropina, beta-endorfina, peptyd diabetogenny, czynnik wytrzeszczowy i hormon wzrostu jajników. Rozważmy bardziej szczegółowo skutki niektórych z nich.

kortykotropina . (hormon adrenokortykotropowy - ACTH) jest wydzielany przez przysadkę mózgową w ciągłych pulsujących impulsach o wyraźnym rytmie dobowym. Wydzielanie kortykotropiny regulowane jest przez działanie bezpośrednie i sprzężenie zwrotne. Bezpośrednie połączenie reprezentuje peptyd podwzgórza - kortykoliberyna, która wzmaga syntezę i wydzielanie kortykotropiny. Sprzężenia zwrotne są wywoływane przez poziomy kortyzolu (hormonu kory nadnerczy) we krwi i są zamykane zarówno na poziomie podwzgórza, jak i przysadki gruczołowej, a wzrost stężenia kortyzolu hamuje wydzielanie kortykoliberyny i kortykotropiny.

Kortykotropina ma dwa rodzaje działania - nadnerczowe i pozanadnerczowe. Działanie nadnerczowe jest główne i polega na stymulacji sekrecji glikokortykoidów, w znacznie mniejszym stopniu mineralokortykosteroidów i androgenów. Hormon wzmaga syntezę hormonów w korze nadnerczy – steroidogenezę i syntezę białek, prowadząc do przerostu i przerostu kory nadnerczy. Działanie pozanadnerczowe polega na lipolizie tkanki tłuszczowej, zwiększonym wydzielaniu insuliny, hipoglikemii, zwiększonym odkładaniu melaniny z przebarwieniami.

Nadmiarowi kortykotropiny towarzyszy rozwój hiperkortyzolizmu z dominującym wzrostem wydzielania kortyzolu i nazywa się to chorobą Itsenko-Cushinga. Główne objawy są typowe dla nadmiaru glikokortykosteroidów: otyłość i inne zmiany metaboliczne, zmniejszenie skuteczności mechanizmów odpornościowych, rozwój nadciśnienia tętniczego i możliwość cukrzycy. Niedobór kortykotropiny powoduje niewydolność funkcji glukokortykoidów nadnerczy z wyraźnymi zmianami metabolicznymi, a także zmniejszenie odporności organizmu na niekorzystne warunki środowiskowe.

Somatotropina . . Hormon wzrostu ma szeroki zakres efektów metabolicznych, które zapewniają efekt morfogenetyczny. Hormon wpływa na metabolizm białek, nasilając procesy anaboliczne. Stymuluje wnikanie aminokwasów do komórek, syntezę białek poprzez przyspieszenie translacji i aktywację syntezy RNA, zwiększa podział komórek i wzrost tkanek oraz hamuje enzymy proteolityczne. Stymuluje wbudowywanie siarczanu do chrząstki, tymidyny do DNA, proliny do kolagenu, urydyny do RNA. Hormon powoduje dodatni bilans azotowy. Stymuluje wzrost chrząstek nasadowych i ich zastępowanie przez tkankę kostną poprzez aktywację fosfatazy alkalicznej.

Wpływ na metabolizm węglowodanów jest dwojaki. Z jednej strony somatotropina zwiększa produkcję insuliny, zarówno dzięki bezpośredniemu wpływowi na komórki beta, jak i hiperglikemii wywołanej przez hormony na skutek rozpadu glikogenu w wątrobie i mięśniach. Somatotropina aktywuje insulinazę wątrobową, enzym rozkładający insulinę. Z drugiej strony somatotropina działa przeciwwyspowo, hamując wykorzystanie glukozy w tkankach. Ta kombinacja efektów, gdy jest predysponowana w warunkach nadmiernego wydzielania, może powodować cukrzycę, zwaną pochodzenia przysadkowego.

Wpływ na metabolizm tłuszczów polega na stymulacji lipolizy tkanki tłuszczowej oraz działaniu lipolitycznym katecholamin, podniesieniu poziomu wolnych kwasów tłuszczowych we krwi; z powodu ich nadmiernego spożycia w wątrobie i utleniania wzrasta tworzenie ciał ketonowych. Te efekty somatotropiny są również klasyfikowane jako diabetogenne.

Jeśli nadmiar hormonu występuje w młodym wieku, powstaje gigantyzm z proporcjonalnym rozwojem kończyn i tułowia. Nadmiar tego hormonu w okresie dojrzewania i dorosłości powoduje wzrost wzrostu nasadowych odcinków kości szkieletu, stref z niepełnym skostnieniem, co nazywa się akromegalią. . Wzrost wielkości i narządów wewnętrznych - splanhomegalia.

Wraz z wrodzonym niedoborem hormonu powstaje karłowatość, nazywana „nanizmem przysadkowym”. Po opublikowaniu powieści J. Swifta o Guliwerze, takich ludzi nazywa się potocznie Lilliputami. W innych przypadkach nabyty niedobór hormonów powoduje łagodne zahamowanie wzrostu.

Prolaktyna . Wydzielanie prolaktyny jest regulowane przez peptydy podwzgórzowe – inhibitor prolaktynostatyny i stymulator prolaktoliberyny. Produkcja neuropeptydów podwzgórza jest pod kontrolą dopaminergiczną. Poziom estrogenu i glukokortykoidów we krwi wpływa na wielkość wydzielania prolaktyny.

i hormony tarczycy.

Prolaktyna specyficznie stymuluje rozwój gruczołu sutkowego i laktację, ale nie jego wydzielanie, które jest stymulowane przez oksytocynę.

Oprócz gruczołów sutkowych prolaktyna wpływa na gruczoły płciowe, pomagając w utrzymaniu aktywności wydzielniczej ciałka żółtego i tworzeniu progesteronu. Prolaktyna jest regulatorem gospodarki wodno-solnej, zmniejsza wydalanie wody i elektrolitów, nasila działanie wazopresyny i aldosteronu, stymuluje wzrost narządów wewnętrznych, erytropoezę, sprzyja manifestacji macierzyństwa. Oprócz wzmagania syntezy białek, zwiększa tworzenie tłuszczu z węglowodanów, przyczyniając się do otyłości poporodowej.

Melanotropina . . Powstaje w komórkach płata pośredniego przysadki mózgowej. Produkcja melanotropiny jest regulowana przez melanoliberynę podwzgórza. Głównym działaniem hormonu jest oddziaływanie na melanocyty skóry, gdzie powoduje depresję pigmentu w procesach, wzrost wolnego pigmentu w naskórku otaczającym melanocyty oraz wzrost syntezy melaniny. Zwiększa pigmentację skóry i włosów.

7. Neurohipofiza, jej związek z podwzgórzem. Wpływ hormonów tylnego przysadki (oksygocyna, ADH). Rola ADH w regulacji objętości płynów w organizmie. Cukrzyca niecukrowa.

Wazopresyna . . Powstaje w komórkach jądra nadwzrokowego i przykomorowego podwzgórza i gromadzi się w neuroprzysadce. Główne bodźce regulujące syntezę wazopresyny w podwzgórzu i jej wydzielanie do krwi przez przysadkę można ogólnie nazwać osmotycznym. Są one reprezentowane przez: a) wzrost ciśnienia osmotycznego osocza krwi i stymulację osmoreceptorów naczyń krwionośnych i neuronów-osmoreceptorów podwzgórza; b) wzrost zawartości sodu we krwi i stymulacja neuronów podwzgórza, które działają jako receptory sodu; c) zmniejszenie centralnej objętości krwi krążącej i ciśnienia tętniczego, postrzegane przez wolomoreceptory serca i mechanoreceptory naczyń;

d) emocjonalny i bolesny stres oraz aktywność fizyczna; e) aktywacja układu renina-angiotensyna i stymulujący wpływ angiotensyny na neurony neurosekrecyjne.

Działanie wazopresyny realizowane jest poprzez wiązanie hormonu w tkankach z dwoma typami receptorów. Wiązanie się z receptorami typu Y1, zlokalizowanymi głównie w ścianach naczyń krwionośnych, poprzez drugie przekaźniki trójfosforan inozytolu i wapń powoduje skurcz naczyń, co przyczynia się do nazwy hormonu – „wazopresyny”. Wiązanie się z receptorami typu Y2 w dystalnym nefronie poprzez drugi przekaźnik cAMP zapewnia wzrost przepuszczalności przewodów zbiorczych nefronu dla wody, jej reabsorpcję i stężenie w moczu, co odpowiada drugiej nazwie wazopresyny – „hormonu antydiuretycznego, ADH".

Oprócz działania na nerki i naczynia krwionośne, wazopresyna jest jednym z ważnych neuropeptydów mózgowych biorących udział w powstawaniu pragnienia i zachowań związanych z piciem, mechanizmach pamięci oraz regulacji wydzielania hormonów przysadkowo-gruczołowych.

Brak lub nawet całkowity brak wydzielania wazopresyny objawia się gwałtownym wzrostem diurezy z uwolnieniem dużej ilości hipotonicznego moczu. Ten zespół nazywa się moczówka prosta", może być wrodzony lub nabyty. Objawia się zespół nadmiaru wazopresyny (zespół Parchona)

w nadmiernym zatrzymaniu płynów w organizmie.

Oksytocyna . Synteza oksytocyny w jądrach przykomorowych podwzgórza i jej uwalnianie do krwi z przysadki mózgowej jest stymulowane drogą odruchową po stymulacji receptorów rozciągania szyjki macicy i gruczołu sutkowego. Estrogeny zwiększają wydzielanie oksytocyny.

Oksytocyna powoduje następujące efekty: a) stymuluje skurcz mięśni gładkich macicy, przyczyniając się do porodu; b) powoduje skurcz komórek mięśni gładkich przewodów wydalniczych gruczołu mlekowego, zapewniając uwalnianie mleka; c) w określonych warunkach ma działanie moczopędne i natriuretyczne; d) uczestniczy w organizacji zachowań związanych z piciem i jedzeniem; e) jest dodatkowym czynnikiem regulującym wydzielanie hormonów przysadkowo-gruczołowych.

8. Kora nadnerczy. Hormony kory nadnerczy i ich funkcja. Regulacja wydzielania kortykosteroidów. Hipo- i nadczynność kory nadnerczy.

Mineralokortykoidy są wydzielane w strefie kłębuszków kory nadnerczy. Głównym mineralokortykoidem jest aldosteron Hormon ten bierze udział w regulacji wymiany soli i wody między środowiskiem wewnętrznym i zewnętrznym, wpływając głównie na aparat kanalikowy nerek, a także gruczoły potowe i ślinowe oraz błonę śluzową jelit. Działając na błony komórkowe sieci naczyniowej i tkanek, hormon reguluje również wymianę sodu, potasu i wody między środowiskiem zewnątrzkomórkowym i wewnątrzkomórkowym.

Główne skutki działania aldosteronu w nerkach to wzrost reabsorpcji sodu w kanalikach dystalnych z jego zatrzymaniem w organizmie oraz wzrost wydalania potasu z moczem przy zmniejszeniu zawartości kationów w organizmie. Pod wpływem aldosteronu dochodzi do opóźnienia w organizmie chlorków, wody, zwiększonego wydalania jonów wodorowych, amonu, wapnia i magnezu. Zwiększa się objętość krwi krążącej, dochodzi do przesunięcia równowagi kwasowo-zasadowej w kierunku zasadowicy. Aldosteron może mieć działanie glukokortykoidowe, ale jest 3 razy słabszy niż kortyzol i nie objawia się w warunkach fizjologicznych.

Mineralokortykoidy są niezbędnymi hormonami, ponieważ śmierci organizmu po usunięciu nadnerczy można zapobiec, wprowadzając hormony z zewnątrz. Mineralokortykoidy nasilają stan zapalny, dlatego czasami nazywane są hormonami przeciwzapalnymi.

Głównym regulatorem powstawania i wydzielania aldosteronu jest angiotensyna II, co umożliwiło rozważenie aldosteronu jako składnika układ renina-angiotensyna-aldosteron (RAAS), zapewnienie regulacji homeostazy wodno-solnej i hemodynamicznej. Sprzężenie zwrotne w regulacji wydzielania aldosteronu realizuje się, gdy zmienia się poziom potasu i sodu we krwi, a także objętość krwi i płynu pozakomórkowego oraz zawartość sodu w moczu kanalików dystalnych.

Nadmierna produkcja aldosteronu – aldosteronizm – może mieć charakter pierwotny i wtórny. W pierwotnym hiperaldosteronizmie nadnercza z powodu przerostu lub guza strefy kłębuszkowej (zespół Kona) wytwarza zwiększone ilości hormonu, co prowadzi do opóźnienia w organizmie sodu, wody, obrzęku i nadciśnienia tętniczego, utraty jony potasu i wodoru przez nerki, zasadowica i zmiany pobudliwości mięśnia sercowego i układu nerwowego. Wtórny hiperaldosteronizm jest wynikiem nadmiernej produkcji angiotensyny II i zwiększonej stymulacji nadnerczy.

Brak aldosteronu w przypadku uszkodzenia nadnerczy przez proces patologiczny jest rzadko izolowany, częściej łączy się z niedoborem innych hormonów substancji korowej. Wiodące zaburzenia obserwuje się w układzie sercowo-naczyniowym i nerwowym, co wiąże się z zahamowaniem pobudliwości,

spadek BCC i przesunięcia w równowadze elektrolitowej.

Glikokortykosteroidy (kortyzol i kortykosteron) ) wpływają na wszystkie rodzaje wymiany.

Hormony mają głównie kataboliczny i antyanaboliczny wpływ na metabolizm białek, powodując ujemny bilans azotowy. rozpad białka następuje w mięśniach, łącznej tkance kostnej, poziom albuminy we krwi spadnie. Zmniejsza się przepuszczalność błon komórkowych dla aminokwasów.

Wpływ kortyzolu na metabolizm tłuszczów wynika z połączenia wpływów bezpośrednich i pośrednich. Synteza tłuszczu z węglowodanów przez sam kortyzol jest hamowana, ale z powodu hiperglikemii wywołanej glikokortykosteroidami i zwiększonego wydzielania insuliny zwiększa się tworzenie tłuszczu. Tłuszcz odkłada się w

górna część ciała, szyja i twarz.

Wpływ na metabolizm węglowodanów jest zasadniczo odwrotny do wpływu insuliny, dlatego glikokortykoidy nazywane są hormonami przeciwwyspowymi. Pod wpływem kortyzolu hiperglikemia występuje na skutek: 1) zwiększonego tworzenia węglowodanów z aminokwasów na drodze glukoneogenezy; 2) hamowanie utylizacji glukozy przez tkanki. Hiperglikemia powoduje glukozurię i stymulację wydzielania insuliny. Zmniejszenie wrażliwości komórek na insulinę wraz z efektami przeciwwyspowymi i katabolicznymi może prowadzić do rozwoju cukrzycy steroidowej.

Działanie ogólnoustrojowe kortyzolu przejawia się w postaci zmniejszenia liczby limfocytów, eozynofili i bazofilów we krwi, wzrostu liczby neutrofili i erytrocytów, wzrostu wrażliwości sensorycznej i pobudliwości układu nerwowego, wzrostu wrażliwości receptorów adrenergicznych na działanie katecholamin, utrzymanie optymalnego stanu funkcjonalnego i regulację układu sercowo-naczyniowego. Glikokortykosteroidy zwiększają odporność organizmu na działanie nadmiernych bodźców oraz tłumią stany zapalne i reakcje alergiczne, dlatego nazywane są hormonami adaptacyjnymi i przeciwzapalnymi.

Nadmiar glikokortykoidów, niezwiązany ze zwiększonym wydzielaniem kortykotropiny, nazywa się Zespół Itsenko-Cushinga. Jej główne objawy są podobne do choroby Itsenko-Cushinga, jednak dzięki sprzężeniu zwrotnemu znacznie zmniejsza się wydzielanie kortykotropiny i jej poziom we krwi. Osłabienie mięśni, skłonność do cukrzycy, nadciśnienie i zaburzenia narządów płciowych, limfopenia, wrzody żołądka, zmiany w psychice - to nie jest pełna lista objawów hiperkortyzolizmu.

Niedobór glikokortykoidów powoduje hipoglikemię, zmniejszoną odporność organizmu, neutropenię, eozynofilię i limfocytozę, upośledzoną aktywność adrenoreaktywności i czynności serca oraz niedociśnienie.

9. Układ współczulno-nadnerczowy, jego organizacja funkcjonalna. Katecholaminy jako mediatory i hormony. Udział w stresie. Nerwowa regulacja tkanki chromochłonnej nadnerczy.

Katecholaminy - hormony rdzenia nadnerczy epinefryna i noradrenalina , które są wydzielane w stosunku 6:1.

główne efekty metaboliczne. adrenaliny to: zwiększony rozpad glikogenu w wątrobie i mięśniach (glikogenoliza) na skutek aktywacji fosforylazy, zahamowanie syntezy glikogenu, zahamowanie zużycia glukozy przez tkanki, hiperglikemia, zwiększone zużycie tlenu przez tkanki i procesy oksydacyjne w nich, aktywacja rozpad i mobilizacja tłuszczu oraz jego utlenianie.

Funkcjonalne działanie katecholamin. zależą od przewagi jednego z typów receptorów adrenergicznych (alfa lub beta) w tkankach. W przypadku adrenaliny główne efekty funkcjonalne przejawiają się w postaci: zwiększonej i zwiększonej częstości akcji serca, poprawy przewodzenia pobudzenia w sercu, zwężenia naczyń skóry i narządów jamy brzusznej; zwiększone wytwarzanie ciepła w tkankach, osłabienie skurczów żołądka i jelit, rozluźnienie mięśni oskrzeli, rozszerzenie źrenic, zmniejszenie filtracji kłębuszkowej i oddawania moczu, pobudzenie wydzielania reniny przez nerki. Tym samym adrenalina powoduje poprawę interakcji organizmu ze środowiskiem zewnętrznym, zwiększa wydolność w stanach nagłych. Adrenalina jest hormonem o naglącej (nagłej) adaptacji.

Uwalnianie katecholamin jest regulowane przez układ nerwowy poprzez włókna współczulne przechodzące przez nerw trzewny. Ośrodki nerwowe regulujące funkcję wydzielniczą tkanki chromochłonnej znajdują się w podwzgórzu.

10. Czynność hormonalna trzustki. Mechanizmy działania jego hormonów na metabolizm węglowodanów, tłuszczów, białek. Regulacja zawartości glukozy w wątrobie, tkance mięśniowej, komórkach nerwowych. Cukrzyca. Hiperinsulinemia.

Hormony regulujące poziom cukru, tj. Wiele hormonów gruczołów dokrewnych wpływa na metabolizm cukru we krwi i węglowodanów. Ale hormony wysp Langerhansa trzustki mają najbardziej wyraźne i silne działanie - insulina i glukagon . Pierwszy z nich można nazwać hipoglikemicznym, ponieważ obniża poziom cukru we krwi, a drugi hiperglikemicznym.

Insulina ma silny wpływ na wszystkie rodzaje metabolizmu. Jego wpływ na metabolizm węglowodanów objawia się głównie następującymi efektami: zwiększa przepuszczalność błon komórkowych w mięśniach i tkance tłuszczowej dla glukozy, aktywuje i zwiększa zawartość enzymów w komórkach, wzmaga wykorzystanie glukozy przez komórki, aktywuje procesy fosforylacji, hamuje rozpad i stymuluje syntezę glikogenu, hamuje glukoneogenezę aktywuje glikolizę.

Główne efekty insuliny na metabolizm białek: zwiększona przepuszczalność błony dla aminokwasów, zwiększona synteza białek niezbędnych do powstania

kwasy nukleinowe, głównie mRNA, aktywacja syntezy aminokwasów w wątrobie, aktywacja syntezy i zahamowanie rozpadu białek.

Główne działanie insuliny na metabolizm tłuszczów: pobudzenie syntezy wolnych kwasów tłuszczowych z glukozy, pobudzenie syntezy trójglicerydów, zahamowanie rozpadu tłuszczów, aktywacja utleniania ciał ketonowych w wątrobie.

Glukagon powoduje następujące główne efekty: aktywuje glikogenolizę w wątrobie i mięśniach, powoduje hiperglikemię, aktywuje glukoneogenezę, lipolizę i hamowanie syntezy tłuszczów, zwiększa syntezę ciał ketonowych w wątrobie, stymuluje katabolizm białek w wątrobie, zwiększa syntezę mocznika.

Głównym regulatorem wydzielania insuliny jest w napływającej krwi D-glukoza, która aktywuje specyficzną pulę cAMP w komórkach beta i poprzez ten mediator prowadzi do stymulacji uwalniania insuliny z ziarnistości wydzielniczych. Wzmacnia odpowiedź komórek beta na działanie glukozy, hormonu jelitowego – żołądkowego peptydu hamującego (GIP). Poprzez nieswoistą, niezależną od glukozy pulę cAMP stymuluje wydzielanie insuliny i jony CA++. Układ nerwowy odgrywa również rolę w regulacji wydzielania insuliny, w szczególności nerw błędny i acetylocholina stymulują wydzielanie insuliny, natomiast nerwy współczulne i katecholaminy hamują wydzielanie insuliny i stymulują wydzielanie glukagonu poprzez receptory alfa-adrenergiczne.

Specyficznym inhibitorem produkcji insuliny jest hormon komórek delta wysepek Langerhansa. - somatostatyna . Hormon ten jest również wytwarzany w jelitach, gdzie hamuje wchłanianie glukozy, a tym samym zmniejsza odpowiedź komórek beta na bodziec glukozowy.

Wydzielanie glukagonu jest stymulowane obniżeniem poziomu glukozy we krwi, pod wpływem hormonów żołądkowo-jelitowych (GIP, gastryna, sekretyna, pankreozymina-cholecystokinina) i obniżeniem zawartości jonów CA++, hamowane przez insulinę, somatostatynę, glukoza i wapń.

Całkowity lub względny brak insuliny w stosunku do glukagonu objawia się cukrzycą, w której dochodzi do głębokich zaburzeń metabolicznych i jeśli nie zostanie sztucznie przywrócona z zewnątrz aktywność insuliny, może dojść do śmierci. Cukrzyca charakteryzuje się hipoglikemią, cukromoczem, wielomoczem, pragnieniem, ciągłym głodem, ketonemią, kwasicą, słabą odpornością, niewydolnością krążenia i wieloma innymi zaburzeniami. Niezwykle ciężkim objawem cukrzycy jest śpiączka cukrzycowa.

11. Tarczyca, fizjologiczna rola jej hormonów. Hipo- i nadczynność.

Hormony tarczycy są trijodotyronina i tetrajodotyronina (tyroksyna ). Głównym regulatorem ich uwalniania jest tyreotropina, hormon adenohypofizy. Ponadto istnieje bezpośrednia regulacja nerwowa tarczycy poprzez nerwy współczulne. Sprzężenie zwrotne zapewnia poziom hormonów we krwi i jest zamknięty zarówno w podwzgórzu, jak i przysadce mózgowej. Intensywność wydzielania hormonów tarczycy wpływa na wielkość ich syntezy w samym gruczole (lokalne sprzężenie zwrotne).

główne efekty metaboliczne. hormonami tarczycy są: zwiększony pobór tlenu przez komórki i mitochondria, aktywacja procesów oksydacyjnych i wzrost podstawowej przemiany materii, stymulacja syntezy białek poprzez zwiększenie przepuszczalności błon komórkowych dla aminokwasów oraz aktywacja aparatu genetycznego komórki, działanie lipolityczne, aktywacja syntezy i wydalania cholesterolu z żółcią, aktywacja rozpadu glikogenu, hiperglikemia, zwiększenie zużycia glukozy przez tkanki, zwiększenie wchłaniania glukozy w jelicie, aktywacja insulinazy wątrobowej i przyspieszenie inaktywacji insuliny, pobudzenie wydzielania insuliny w wyniku hiperglikemii.

Główne działanie funkcjonalne hormonów tarczycy to: zapewnienie prawidłowych procesów wzrostu, rozwoju i różnicowania tkanek i narządów, aktywacja działania współczulnego poprzez zmniejszenie rozpadu mediatora, tworzenie metabolitów podobnych do katecholamin oraz zwiększenie wrażliwości receptorów adrenergicznych ( tachykardia, pocenie się, skurcz naczyń itp.), zwiększenie wytwarzania ciepła i temperatury ciała, aktywacja GNI i zwiększona pobudliwość ośrodkowego układu nerwowego, zwiększona wydajność energetyczna mitochondriów i kurczliwość mięśnia sercowego, działanie ochronne w stosunku do rozwoju uszkodzeń i owrzodzeń mięśnia sercowego w żołądku w stresie, zwiększony przepływ krwi przez nerki, filtracja kłębuszkowa i diureza, stymulacja procesów regeneracji i gojenia, zapewniająca normalną aktywność rozrodczą.

Zwiększone wydzielanie hormonów tarczycy jest przejawem nadczynności tarczycy - nadczynności tarczycy. Jednocześnie odnotowuje się charakterystyczne zmiany w metabolizmie (zwiększony metabolizm podstawowy, hiperglikemia, utrata masy ciała itp.), Objawy nadmiernego działania współczulnego (tachykardia, zwiększone pocenie się, zwiększona pobudliwość, podwyższone ciśnienie krwi itp.). Może

rozwijać cukrzycę.

Wrodzony niedobór hormonów tarczycy zaburza wzrost, rozwój i różnicowanie kośćca, tkanek i narządów, w tym układu nerwowego (występuje upośledzenie umysłowe). Ta wrodzona patologia nazywa się „kretynizmem”. Nabyta niewydolność tarczycy lub niedoczynność tarczycy objawia się spowolnieniem procesów oksydacyjnych, zmniejszeniem podstawowej przemiany materii, hipoglikemią, zwyrodnieniem podskórnej tkanki tłuszczowej i skóry z nagromadzeniem glikozaminoglikanów i wody. Zmniejsza się pobudliwość ośrodkowego układu nerwowego, osłabia działanie współczulne i wytwarzanie ciepła. Kompleks takich naruszeń nazywa się „obrzękiem śluzowatym”, tj. obrzęk śluzówki.

Kalcytonina - wytwarzane w parafolikularnych komórkach K tarczycy. Narządami docelowymi dla kalcytoniny są kości, nerki i jelita. Kalcytonina obniża poziom wapnia we krwi, ułatwiając mineralizację i hamując resorpcję kości. Zmniejsza wchłanianie zwrotne wapnia i fosforanów w nerkach. Kalcytonina hamuje wydzielanie gastryny w żołądku i zmniejsza kwasowość soku żołądkowego. Wydzielanie kalcytoniny jest stymulowane wzrostem poziomu Ca ++ we krwi oraz gastryną.

12. Przytarczyce, ich rola fizjologiczna. Mechanizmy konserwacji

stężenia wapnia i fosforanów we krwi. Wartość witaminy D.

Regulacja metabolizmu wapnia odbywa się głównie dzięki działaniu paratyryny i kalcytoniny, w przytarczycach syntetyzuje się parathormon, czyli paratyryna, hormon przytarczyc. Zapewnia wzrost poziomu wapnia we krwi. Narządami docelowymi dla tego hormonu są kości i nerki. W tkance kostnej para-tyryna wzmacnia funkcję osteoklastów, co przyczynia się do demineralizacji kości oraz wzrostu poziomu wapnia i fosforu w osoczu krwi. W aparacie kanalikowym nerek paratyryna stymuluje wchłanianie zwrotne wapnia i hamuje wchłanianie zwrotne fosforanów, co prowadzi do hiperkalcemii i fosfaturii. Pewne znaczenie w realizacji hiperkalcemicznego działania hormonu może mieć rozwój fosfaturii. Wynika to z faktu, że wapń tworzy nierozpuszczalne związki z fosforanami; dlatego zwiększone wydalanie fosforanów z moczem przyczynia się do wzrostu poziomu wolnego wapnia w osoczu krwi. Parathyrin nasila syntezę kalcytriolu, który jest aktywnym metabolitem witaminy D 3 . Ta ostatnia najpierw powstaje w stanie nieaktywnym w skórze pod wpływem promieniowania ultrafioletowego, a następnie pod wpływem paratyryny jest aktywowana w wątrobie i nerkach. Kalcytriol wzmaga tworzenie białka wiążącego wapń w ścianie jelita, co sprzyja reabsorpcji wapnia i rozwojowi hiperkalcemii. Zatem wzrost reabsorpcji wapnia w jelicie podczas nadprodukcji paratyryny wynika głównie z jej stymulującego działania na aktywację witaminy D3. Bezpośredni wpływ samej paratyryny na ścianę jelita jest bardzo nieznaczny.

Po usunięciu przytarczyc zwierzę umiera z powodu drgawek tężcowych. Wynika to z faktu, że w przypadku niskiej zawartości wapnia we krwi gwałtownie wzrasta pobudliwość nerwowo-mięśniowa. Jednocześnie działanie nawet niewielkich bodźców zewnętrznych prowadzi do skurczu mięśni.

Nadprodukcja paratyryny prowadzi do demineralizacji i resorpcji tkanki kostnej, rozwoju osteoporozy. Poziom wapnia w osoczu krwi gwałtownie wzrasta, w wyniku czego wzrasta tendencja do tworzenia kamieni w narządach układu moczowo-płciowego. Hiperkalcemia przyczynia się do rozwoju wyraźnych zaburzeń stabilności elektrycznej serca, a także powstawania wrzodów w przewodzie pokarmowym, których występowanie wynika ze stymulującego działania jonów Ca 2+ na produkcję gastryny i chlorowodoru kwas w żołądku.

Wydzielanie paratyryny i tyrokalcytoniny (patrz punkt 5.2.3) jest regulowane przez rodzaj ujemnego sprzężenia zwrotnego w zależności od poziomu wapnia w osoczu krwi. Wraz ze spadkiem zawartości wapnia zwiększa się wydzielanie paratyryny i hamowana jest produkcja tyrokalcytoniny. W warunkach fizjologicznych można to zaobserwować podczas ciąży, laktacji, obniżonej zawartości wapnia w przyjmowanym pokarmie. Przeciwnie, wzrost stężenia wapnia w osoczu krwi pomaga zmniejszyć wydzielanie paratyryny i zwiększyć produkcję tyrokalcytoniny. Ta ostatnia może mieć ogromne znaczenie u dzieci i młodzieży, ponieważ w tym wieku odbywa się tworzenie szkieletu kostnego. Odpowiedni przebieg tych procesów jest niemożliwy bez tyrokalcytoniny, która warunkuje wchłanianie wapnia z osocza krwi i włączanie go w strukturę tkanki kostnej.

13. Gruczoły płciowe. Funkcje żeńskich hormonów płciowych. Cykl miesiączkowo-jajnikowy, jego mechanizm. Zapłodnienie, ciąża, poród, laktacja. Regulacja hormonalna tych procesów. Związane z wiekiem zmiany w produkcji hormonów.

męskie hormony płciowe .

Męskie hormony płciowe - androgeny - powstaje w komórkach Leydiga jąder z cholesterolu. Głównym ludzkim androgenem jest testosteron . . W korze nadnerczy wytwarzane są niewielkie ilości androgenów.

Testosteron ma szerokie działanie metaboliczne i fizjologiczne: zapewnia procesy różnicowania w embriogenezie oraz rozwoju pierwotnych i wtórnych cech płciowych, tworzenie struktur OUN zapewniających zachowania seksualne i funkcje seksualne, uogólniony efekt anaboliczny zapewniający wzrost kośćca i mięśni, rozmieszczenie tłuszczu podskórnego, zapewnienie spermatogenezy, zatrzymywanie azotu, potasu, fosforanów w organizmie, aktywacja syntezy RNA, stymulacja erytropoezy.

Androgeny powstają również w niewielkich ilościach w kobiecym ciele, będąc nie tylko prekursorami syntezy estrogenów, ale także wspierając popęd seksualny, a także stymulując wzrost włosów łonowych i pod pachami.

żeńskie hormony płciowe .

Wydzielanie tych hormonów estrogen) jest ściśle związany z cyklem rozrodczym samicy. Cykl płciowy kobiety zapewnia wyraźną integrację w czasie różnych procesów niezbędnych do realizacji funkcji rozrodczej - okresowe przygotowanie endometrium do implantacji zarodka, dojrzewanie jaja i owulacja, zmiany drugorzędowych cech płciowych itp. Koordynacja tych procesy zapewniają wahania wydzielania szeregu hormonów, przede wszystkim gonadotropin i sterydów płciowych. Wydzielanie gonadotropin odbywa się „tonicznie”, tj. w sposób ciągły i „cyklicznie”, z okresowym uwalnianiem dużych ilości follikuliny i luteotropiny w połowie cyklu.

Cykl płciowy trwa 27-28 dni i dzieli się na cztery okresy:

1) przedowulacyjny - okres przygotowań do ciąży, macica w tym czasie powiększa się, rośnie błona śluzowa i jej gruczoły, nasilają się i stają się częstsze skurcze jajowodów i warstwy mięśniowej macicy, błona śluzowa pochwy również rośnie;

2) owulacyjny- rozpoczyna się pęknięciem pęcherzykowego pęcherzyka jajnikowego, uwolnieniem z niego komórki jajowej i jej przejściem przez jajowód do jamy macicy. W tym okresie zwykle dochodzi do zapłodnienia, cykl seksualny zostaje przerwany i dochodzi do ciąży;

3) po owulacji- u kobiet w tym okresie pojawia się miesiączka, niezapłodnione jajo, które pozostaje żywe w macicy przez kilka dni, umiera, toniczne skurcze mięśni macicy nasilają się, co prowadzi do odrzucenia jej błony śluzowej i uwolnienia skrawków śluz wraz z krwią.

4) okres odpoczynku- występuje po zakończeniu okresu poowulacyjnego.

Przesunięciom hormonalnym podczas cyklu płciowego towarzyszą następujące rearanżacje. W okresie przedowulacyjnym najpierw następuje stopniowy wzrost wydzielania folitropiny przez przysadkę mózgową. Dojrzewający pęcherzyk wytwarza coraz większą ilość estrogenów, które w sprzężeniu zwrotnym zaczynają zmniejszać produkcję follinotropiny. Rosnący poziom lutropiny prowadzi do stymulacji syntezy enzymów, co prowadzi do ścieńczenia ściany pęcherzyka niezbędnego do owulacji.

W okresie owulacji następuje gwałtowny wzrost poziomu lutropiny, folitropiny i estrogenu we krwi.

W początkowej fazie okresu poowulacyjnego następuje krótkotrwały spadek poziomu gonadotropin i estradiol pęknięty pęcherzyk zaczyna wypełniać się komórkami lutealnymi, tworzą się nowe naczynia krwionośne. Zwiększenie produkcji progesteron utworzone przez ciałko żółte zwiększa wydzielanie estradiolu przez inne dojrzewające pęcherzyki. Wynikający z tego poziom progesteronu i estrogenu w sprzężeniu zwrotnym hamuje wydzielanie follotropiny i luteotropiny. Rozpoczyna się degeneracja ciałka żółtego, spada poziom progesteronu i estrogenów we krwi. W nabłonku wydzielniczym bez stymulacji sterydami dochodzi do zmian krwotocznych i zwyrodnieniowych, które prowadzą do krwawienia, odrzucenia śluzówki, skurczu macicy tj. do menstruacji.

14. Funkcje męskich hormonów płciowych. regulacja ich edukacji. Przed- i postnatalne działanie hormonów płciowych na organizm. Związane z wiekiem zmiany w produkcji hormonów.

Funkcja hormonalna jąder.

1) Komórki Sertollego – produkują hormon-inhibinę – hamują powstawanie folitropiny w przysadce mózgowej, powstawanie i wydzielanie estrogenów.

2) Komórki Leydiga – produkują hormon testosteron.

  1. Zapewnia procesy różnicowania w embriogenezie
  2. Rozwój pierwotnych i wtórnych cech płciowych
  3. Tworzenie struktur OUN zapewniających zachowania i funkcje seksualne
  4. Działanie anaboliczne (wzrost kośćca, mięśni, dystrybucja tłuszczu podskórnego)
  5. Regulacja spermatogenezy
  6. Zatrzymuje w organizmie azot, potas, fosforany, wapń
  7. Aktywuje syntezę RNA
  8. Stymuluje erytropoezę.

Funkcja endokrynologiczna jajników.

W ciele kobiety hormony są wytwarzane w jajnikach, a komórki warstwy ziarnistej pęcherzyków wytwarzające estrogeny (estradiol, estron, estriol) i komórki ciałka żółtego (progesteron) pełnią funkcję hormonalną.

Funkcje estrogenu:

  1. Zapewniają zróżnicowanie płciowe w embriogenezie.
  2. Dojrzewanie i rozwój cech płciowych kobiet
  3. Ustalenie kobiecego cyklu płciowego, wzrost mięśni macicy, rozwój gruczołów sutkowych
  4. Określ zachowania seksualne, oogenezę, zapłodnienie i implantację w jajach
  5. Rozwój i różnicowanie płodu oraz przebieg aktu urodzeń
  6. Tłumią resorpcję kości, zatrzymują azot, wodę, sole w organizmie

Funkcje progesteronu:

1. Hamuje skurcze mięśni macicy

2. Potrzebny do owulacji

3. Hamuje wydzielanie gonadotropiny

4. Działa antyaldosteronowo, czyli stymuluje natriurezę.

15. Grasica (grasica), jego rola fizjologiczna.

Grasica jest również nazywana grasicą lub grasicą. Podobnie jak szpik kostny jest centralnym narządem immunogenezy (tworzenie odporności). Grasica znajduje się bezpośrednio za mostkiem i składa się z dwóch płatów (prawego i lewego), połączonych luźnym włóknem. Grasica powstaje wcześniej niż inne narządy układu odpornościowego, jej masa u noworodków wynosi 13 g, największą masę - około 30 g - ma grasica u dzieci w wieku 6-15 lat.

Następnie ulega ona odwrotnemu rozwojowi (inwolucji wieku) i u dorosłych zostaje prawie całkowicie zastąpiona przez tkankę tłuszczową (u osób po 50. roku życia tkanka tłuszczowa stanowi 90% całkowitej masy grasicy (średnio 13-15 g)). Okres najintensywniejszego wzrostu organizmu związany jest z aktywnością grasicy. Grasica zawiera małe limfocyty (tymocyty). Decydująca rola grasicy w tworzeniu układu odpornościowego stała się jasna dzięki eksperymentom przeprowadzonym przez australijskiego naukowca D. Millera w 1961 roku.

Odkrył, że usunięcie grasicy z nowonarodzonych myszy spowodowało zmniejszenie produkcji przeciwciał i wydłużenie życia przeszczepionej tkanki. Fakty te wskazują, że grasica bierze udział w dwóch formach odpowiedzi immunologicznej: w reakcjach typu humoralnego – wytwarzaniu przeciwciał oraz w reakcjach typu komórkowego – odrzuceniu (śmierci) przeszczepionej tkanki obcej (przeszczepu), która zachodzi przy udziale różnych klas limfocytów. Tak zwane limfocyty B odpowiadają za produkcję przeciwciał, a limfocyty T odpowiadają za reakcje odrzucenia przeszczepu. Limfocyty T i B powstają w wyniku różnych przemian komórek macierzystych szpiku kostnego.

Przenikając z niej do grasicy, komórka macierzysta pod wpływem hormonów tego narządu przekształca się najpierw w tzw. tymocyt, a następnie, dostając się do śledziony lub węzłów chłonnych, w immunologicznie czynny limfocyt T. Przekształcenie komórki macierzystej w limfocyt B zachodzi najwyraźniej w szpiku kostnym. W grasicy wraz z powstawaniem limfocytów T z komórek macierzystych szpiku kostnego wytwarzane są czynniki hormonalne - tymozyna i tymopoetyna.

Hormony, które zapewniają różnicowanie (różnicę) limfocytów T i odgrywają rolę w komórkowej odpowiedzi immunologicznej. Istnieją również dowody na to, że hormony zapewniają syntezę (konstrukcję) niektórych receptorów komórkowych.

Gruczoły dokrewne są również określane jako gruczoły dokrewne lub dokrewne. Gruczoły dokrewne wydzielają hormony. Gruczoły zawdzięczają swoją nazwę brakowi przewodów wydalniczych. Wytwarzane przez nie substancje czynne zaczynają być uwalniane do krwi.

Ludzkie gruczoły dokrewne obejmują:

  • Nadnercza.
  • Trzustka.
  • Układ podwzgórzowo-przysadkowy.
  • grasica.
  • Epifiza
  • Gruczoły płciowe.

Krótki opis

Poniższa tabela zawiera ogólny opis tak zwanych gruczołów dokrewnych.

NazwaOpis
PrzysadkaJest to główny gruczoł. Zapewnia uwalnianie hormonów regulujących pracę innych gruczołów.
nadnerczaKora i rdzeń to różne koncepcje.
przytarczyceLudzie mają 4 przytarczyce.
Część endokrynna trzustkiJego komórki stanowią nie więcej niż 1 procent całości. Pozostałe komórki pełnią funkcję gruczołów wydzielania zewnętrznego.
grasicaPełni funkcje narządu odporności.
Część endokrynna gonadU kobiet są to jajniki, u mężczyzn jądra.
ŁożyskoPokazuje aktywność podczas ciąży.

Cechy podwzgórza

W swojej anatomicznej naturze nie należy do gruczołów dokrewnych. Obejmuje komórki nerwowe, które syntetyzują hormony we krwi.

Formacje jądrowe regionu podwzgórza biorą udział w utrzymaniu prawidłowej temperatury ciała. W strefie przedoptycznej znajdują się neurony odpowiedzialne za monitorowanie temperatury krwi.

Należy również wymienić inne funkcje podwzgórza:

  • regulacja funkcji układu sercowego;
  • regulacja funkcji układu naczyniowego;
  • regulacja bilansu wodnego;
  • regulacja czynności skurczowej macicy;
  • regulacja aktywności behawioralnej;
  • tworzenie uczucia głodu i sytości.

Najczęstszą zmianą podwzgórza jest prolactinoma. Najczęściej występuje u kobiet. Dzięki temu hormonalnie aktywnemu guzowi zaczyna się wytwarzać. U osób obojga płci diagnozowana jest kolejna poważna patologia.

Cechy przysadki mózgowej

Nazywa się mały gruczoł, którego masa waha się od 0,5 do 0,7 grama. Znajduje się w dole przysadki tureckiego siodła kości klinowej. Hormon ten składa się z płatów przedniego, środkowego i tylnego.

Płat przedni wydziela następujące substancje:

  • Somatotropowy.
  • Gonadotropowy.

Duże znaczenie ma hormon somatotropowy, który kontroluje procesy metaboliczne, a także kontroluje wzrost mięśni i kości. Stymulator tarczycy ma kontrolować tarczycę. Substancja adrenokortykotropowa kontroluje pracę kory nadnerczy.

Niedobór przysadki prowadzi do. Lekarze uważają, że taka choroba jest nie mniej niebezpieczna niż cukrzyca. Nadmiar prowadzi do zaburzeń miesiączkowania u kobiet i impotencji u mężczyzn.

Cechy narządu dokrewnego tarczycy

Ogromną rolę w ludzkim ciele odgrywa narząd dokrewny tarczycy, który przyczynia się do uwalniania następujących zawierających jod:

  • tyroksyna;
  • terokalcytonina;
  • trijodotyronina.

Wytwarzane przez nią substancje kontrolują fosfor, metabolizm wapnia, a także poziom kosztów energii, z których większość jest niezbędna dla organizmu. Gruczoły przytarczyczne wydzielają hormony, które zwiększają poziom wapnia i fosforu we krwi.

Normalne funkcjonowanie „gruczołu tarczycy”, a także jego produktywność, odbywa się dzięki regularnemu przyjmowaniu do organizmu 200 mikrogramów jodu. Ludzie dostają to z jedzeniem, płynami, powietrzem. Niedoczynność gruczołu może prowadzić do niedoczynności tarczycy. Młode kobiety z niedostateczną funkcją tarczycy często rozwijają zaburzenia obsesyjno-kompulsyjne. Na tym tle wiele dziewcząt rozwija się w depresję.

Niedobór wpływa niekorzystnie na stan układu naczyniowego i sercowego. Normalne funkcjonowanie serca zostaje zakłócone i na tym tle rozwija się niewydolność serca. 30 procent pacjentów ma niskie ciśnienie krwi.

Cechy nadnerczy

Hormony w nadnerczach wytwarzają korę i rdzeń. W substancji korowej przeprowadza się syntezę kortykosteroidów. Ponadto hormony wytwarzane są przez następujące strefy:

  • kłębuszkowy;
  • Belka;
  • siatka.

W strefie kłębuszkowej kontrolowana jest nie tylko produkcja mineralokortykoidów, dezoksykortykosteronu, ale także ich metabolizm mineralny. W strefie wiązki odbywa się produkcja glukokortykoidów, kortyzolu i kortykosteronu. Kontroluje również metabolizm tłuszczów, węglowodanów i białek.

W strefie siatkowatej wytwarzane są androgeny i hormony płciowe. Rdzeń jest dostawcą i. Adrenalina odpowiada za pozytywne emocje. Norepinefryna kontroluje procesy nerwowe.

Cechy trzustki

Do mieszanych gruczołów lekarze zaliczają trzustkę. Znajduje się w jamie brzusznej, na poziomie trzonów jednego lub dwóch kręgów lędźwiowych za żołądkiem.

Od żołądka żelazo jest chronione workiem wypełniającym. Średnia waga dorosłego gruczołu waha się od osiemdziesięciu do stu gramów. Długość waha się od czternastu do osiemnastu, grubość - od dwóch do trzech, szerokość - od trzech do dziewięciu centymetrów.

Ten gruczoł pełni niejednoznaczną funkcję. Jego określone komórki wytwarzają sok trawienny. Wchodzi do jelita przez przewody wydalnicze. Inne komórki biorą udział w produkcji insuliny, która odpowiada za przekształcanie nadmiaru glukozy w glikogen. Pomaga to obniżyć poziom cukru we krwi. Niedobór insuliny może prowadzić do rozwoju cukrzycy.

Wyróżnia się również tutaj, który jest antagonistą insuliny. Produkcja somatostatyny prowadzi do zahamowania syntezy glukagonu, insuliny i hormonu wzrostu.

Gruczoły mieszane obejmują również jądra i jajniki. Należą do gonad, które pełnią funkcje zewnątrzwydzielnicze i wewnątrzwydzielnicze. Zakłada się powstawanie i uwalnianie plemników i komórek jajowych, a także odpowiedzialność za produkcję hormonów płciowych.

Jajniki są odpowiedzialne za realizację procesów endokrynnych i generatywnych. Znajdują się w okolicy miednicy. Ich długość waha się od dwóch do pięciu centymetrów. Masa jajników waha się od pięciu do ośmiu gramów. Szerokość jajników waha się od dwóch do dwóch i pół centymetra.

Jajniki odpowiadają również za dojrzewanie jaj i produkcję:

  • progesteron.

Następuje zmiękczenie szyjki macicy, co przyczynia się do pomyślnego rozwiązania obciążenia.

Jądra zlokalizowane w mosznie odpowiadają za funkcje endokrynologiczne i generatywne. Odpowiadają za powstawanie i dojrzewanie plemników. Biorą również udział w tworzeniu testosteronu.

Serce, nerki i CNS

Najważniejszą częścią układu hormonalnego są nerki. Ważną rolę odgrywa „silnik” człowieka, serce, a także centralny układ nerwowy. Nerki pełnią funkcje wydalnicze i hormonalne. Synteza reniny odbywa się za pomocą aparatu przykłębuszkowego. Renina odpowiada za regulację napięcia naczyniowego. Ponadto nerki odpowiadają za syntezę erytroetyny. Odpowiada za czerwone krwinki w szpiku kostnym.

W atrium prowadzona jest produkcja. Serce wpływa również na produkcję sodu przez nerki.

Najważniejszymi hormonami układu nerwowego i hormonalnego są enkefaliny. Ich synteza odbywa się w ośrodkowym układzie nerwowym. Ich główną funkcją jest łagodzenie bólu. Z tego powodu określa się je również mianem opiatów endogennych. Działanie neurohormonów jest podobne do działania morfiny.

Cechy gruczołów wydzielania zewnętrznego

Ważną rolę odgrywają gruczoły zewnątrzwydzielnicze. To gruczoły wydzielania zewnętrznego wydzielają różnorodne substancje na powierzchni ciała, a także do wewnętrznego środowiska ludzkiego ciała. Odpowiadają za kształtowanie się gatunku i indywidualnego aromatu. Kolejną ważną funkcją jest ochrona organizmu przed wnikaniem szkodliwych drobnoustrojów. Ich sekret ma działanie bakteriobójcze i mykostatyczne.

cztery gruczoły

Zewnętrzne gruczoły wydzielnicze obejmują:

  • mleczarnia;
  • pot;
  • ślina i łza.

Są bezpośrednio zaangażowani w regulację zarówno relacji międzygatunkowych, jak i wewnątrzgatunkowych.

Za co są odpowiedzialni?

Gruczoły ślinowe są małe i duże. Znajdują się w ustach człowieka. W błonie podśluzowej znajdują się małe gruczoły. Główne gruczoły ślinowe to sparowane narządy zlokalizowane poza jamą ustną.

Przebieg procesów wydzielniczych odbywa się zwykle w okresie aktywności procesów hormonalnych. Głównym wyzwalaczem jest restrukturyzacja hormonalna. Największą intensywność procesów wydzielniczych obserwuje się bliżej okresu dojrzewania.

Gruczoły sutkowe prezentowane są w postaci przekształconych gruczołów potowych. Ich układanie odbywa się po 6-7 tygodniach. Na początku są jak foki w naskórku. Następnie dochodzi do powstawania punktów mlecznych. Przed okresem dojrzewania gruczoły sutkowe są nieaktywne. Chłopcy i dziewczęta rozwijają się inaczej.

Za produkcję potu odpowiedzialne są gruczoły potowe zaangażowane w proces termoregulacji. Reprezentowane są przez najprostsze rurki, których końce są złożone.

Wniosek

Radykalny brak któregokolwiek z gruczołów może prowadzić do zakłócenia funkcjonowania pozostałych. Czasami dochodzi do śmierci. Dziś za pomocą silnych leków można przeprowadzić jedynie wymianę hormonów tarczycy.

Bibliografia

  1. Nadciśnienie tętnicze u kobiet w ciąży Stan przedrzucawkowy (stan przedrzucawkowy). Makarov O.V., Volkova E.V. RASPM; Moskwa; TsKMS GOU VPO RGMU.-31 s.- 2010.
  2. Nowy miód. technologia (Zalecenia metodyczne) „Postępowanie w ciąży przedwczesnej powikłanej przedwczesnym pęknięciem błon płodowych”; Makarov O.V., Kozlov P.V. (Edycja Volodin N.N.) - RASPM; Moskwa; TsKMS GOU VPO RSMU-2006.
  3. Anomalie aktywności zawodowej: przewodnik dla lekarzy. Certyfikacja UMO dla edukacji medycznej. Podtetenev AD, Strizhova N.V. 2006 Wydawca: MIA.
  4. Opieka w nagłych wypadkach w położnictwie i ginekologii: krótki przewodnik. Sierow V.N. 2008 Wydawca: Geotar-Media.
  5. Ciąża pozamaciczna. Certyfikacja UMO dla edukacji medycznej. Sidorova I.S., Guriev T.D. 2007 Wydawca: Medycyna Praktyczna
  6. Ciąża nie rozwijająca się. Radzinsky V.E., Dimitrova V.I., Mayskova I.Yu. 2009 Wydawca: Geotar-Media.

Umieszczenie akcentu: TAJEMNICA WEWNĘTRZNA

WYDZIELANIE WEWNĘTRZNE (łac. secretio - wydzielanie) - zdolność pewnej grupy gruczołów ludzkich i zwierzęcych (gruczoły dokrewne, muł "gruczoły dokrewne) do wydzielania określonych produktów ich życiowej aktywności ( hormony) bezpośrednio do krwi lub płynu tkankowego, a nie do środowiska zewnętrznego (jak na przykład gruczoły potowe), a nie do jamy narządów wewnętrznych (na przykład gruczołów przewodu żołądkowo-jelitowego). żołądź Vs. Są to: przysadka, tarczyca, sparowane przytarczyce (przytarczyce), nadnercza, gonady męskie (jądra) i żeńskie (jajniki) (ich elementy wewnątrzwydzielnicze). Organ B.

Z. jest również aparatem wysepkowym (oddziałem) trzustki. Do gruczołów dokrewnych zalicza się również wole, czyli grasicę, gruczoł (grasicę) i szyszynkę (szyszynę), chociaż przynależności tych formacji do gruczołów dokrewnych nie można obecnie uznać za ściśle udowodnioną.

Specyficzne substancje biologicznie czynne wydzielane przez gruczoły V. s. - hormony, wchodzące do krwi, są przenoszone w całym ciele i zmieniają metabolizm i energię, aktywność układu nerwowego i narządów wewnętrznych, stymulując lub hamując ich pracę. Hormony wpływają na wzrost fizyczny. i psychiczny. rozwój, dojrzewanie, rozwój drugorzędowych cech płciowych, pigmentacja, wydzielanie mleka, zmiana napięcia mięśni gładkich, aktywacja wzrostu i różnicowania tkanek i narządów.

Oprócz konkretnych wpływ na aktywność enzymów, witamin oraz na niektóre rodzaje metabolizmu (węglowodany, białka, tłuszcze, minerały), każdy gruczoł wraz ze swoimi hormonami ma w różnym stopniu wpływ (bezpośredni lub pośredni) na inne rodzaje metabolizmu. Przysadka mózgowa wytwarza tzw. hormony tronowe stymulujące aktywność innych gruczołów V. s. (gonadotropowy - stymulujący gruczoły płciowe, tyreotropowy - aktywujący funkcję tarczycy itp.). Tak więc stan funkcjonalny wszystkich gruczołów V. s. a ich wpływ na organizm są ze sobą ściśle powiązane. Reprezentują jedną fizjologię system, w regulacji aktywności cięcie odgrywa zasadniczą rolę ośrodkowego układu nerwowego. Ze swojej strony gruczoły V. z. wywierają pewien wpływ na aktywność układu nerwowego, będąc ważnym ogniwem w jednym systemie neurohumoralnej regulacji funkcji w organizmie. Wszystko to świadczy o tym, że gruczoły strony V. są. wydzielane przez nie hormony uczestniczące w regulacji procesów życiowych na wszystkich etapach rozwoju, w tym w okresie embrionalnym, okresie intensywnego wzrostu organizmu i jego dojrzewania, a także w procesie życiowej aktywności dojrzałego organizmu, odgrywają ważną rolę w jej tworzeniu i regulacji czynności różnych narządów i układów funkcjonalnych.

Pomimo tego, że gruczoły V. strony. są ze sobą w ścisłym związku, a porażce jednego gruczołu zwykle towarzyszy naruszenie funkcji innych gruczołów, choroby poszczególnych gruczołów V. s. powodują objawy, charakterystyczne dla porażki każdego z nich, co pozwala określić je jako niezależne choroby, żyto nazywa się endokrynnymi. Naruszenia czynności gruczołów dokrewnych są dwojakiego rodzaju: a) zwiększona aktywność gruczołu - nadczynność, z nacięciem, powstaje zwiększona ilość hormonu i uwalniana do krwi oraz b) osłabienie aktywności gruczołu - niedoczynność gdy zmniejszona ilość hormonu powstaje i jest uwalniana do krwi.

Wraz z porażką przysadki mózgowej, która dzieli się na płaty przedni (gruczołowy), środkowy i tylny (nerwowy), rozwija się szereg chorób. Nadczynność przedniego płata przysadki we wczesnym wieku, kiedy organizm jeszcze rośnie, w niektórych przypadkach prowadzi (z powodu nadmiernej produkcji tzw. hormonu wzrostu) do rozwoju gigantyzm: wzrost takich osób może osiągnąć 2,5 - 2,6 m, wzrasta wzrost zewnętrznych narządów płciowych (z osłabieniem pożądania seksualnego). Jeśli taka nadczynność (z guzem, przewlekłym stanem zapalnym) wystąpi pod koniec wzrostu, może się rozwinąć akromegalia(przyrost dłoni i jęków, łuki brwiowe, kości policzkowe, szczęki itp.). W przypadku niektórych guzów przedniego płata przysadki, pełność wzrasta, na ciele pojawiają się niebiesko-fioletowe paski bliznowate (rozstępy), wzrasta ciśnienie krwi, u kobiet zanika miesiączka, a czasami pojawiają się objawy cukrzycy ( Choroba Itsenko-Cushinga). Z niedoczynnością przedniego płata przysadki we wczesnym dzieciństwie (w wyniku niewystarczającego wytwarzania hormonu wzrostu) rozwija się nanizm (wzrost karłowaty); wzrost kości i rozwój narządów płciowych są zawieszone, metabolizm jest zmniejszony, drugorzędowe cechy płciowe nie rozwijają się.

Przy niewystarczającym wytwarzaniu hormonów „tropikalnych” w przednim płacie przysadki mózgowej aktywność odpowiednich innych gruczołów gruczołu komorowego słabnie. zmniejsza się zdolność przystosowania się organizmu do szkodliwych wpływów. Z uszkodzeniem tylnego płata przysadki mózgowej lub powiązanych działów podwzgórza. w okolicy mózgu pojawia się zwiększone pragnienie (pacjenci piją do 10-15 litrów wody dziennie), a zatem gwałtownie wzrasta oddawanie moczu ( moczówka prosta). Przy całkowitym uszkodzeniu przysadki, silnym wyczerpaniu, gwałtownej utracie wagi, rozwoju osłabienia, wypadaniu zębów itp. ( kacheksja przysadkowa).

Uszkodzenie tarczycy prowadzi z jej nadczynnością do tyreotoksykozy (choroba Gravesa-Basedowa). Wraz z nadczynnością i atrofią tego gruczołu, która występuje we wczesnym dzieciństwie, rozwija się kretynizm, któremu towarzyszy opóźnienie wzrostu, upośledzenie umysłowe, czasami osiągając idiotyzm. Niedoczynność tarczycy w późniejszym wieku prowadzi do obrzęku śluzowatego. Lekko i wcześnie formy nadczynności lub niedoczynności tarczycy są zwykle nazywane (odpowiednio) nadczynnością lub niedoczynnością tarczycy. W miejscach, gdzie w wodzie brakuje jodu, który wchodzi w skład hormonu tarczycy - tyroksyny, często rozwija się wole endemiczne.

Przy nadmiernym wytwarzaniu hormonu przytarczyc (na przykład z guzem) występuje choroba szkieletu kostnego - osteodystrofia przytarczyc, charakteryzujący się niezwykłą miękkością i kruchością kości. Przy niedoczynności przytarczyc rozwija się tężyczka, krawędzie u ludzi (częściej u dzieci, kobiet w ciąży i matek karmiących) wyrażają się w pojawieniu się skurczów mięśni kończyn, twarzy, gardła; ręce podczas napadów konwulsyjnych są ściśnięte - zredukowane. Niewydolność przytarczyc prowadzi również (zwłaszcza w młodym wieku) do próchnicy zębów, wczesnego wypadania włosów i utraty wagi.

Wśród chorób nadnerczy najczęściej występują 2 formy: choroba brązowa(najczęściej spowodowane obustronną gruźlicą nadnerczy), przy skaleczeniu głównymi objawami są pigmentacja skóry i silne osłabienie mięśni (adynamia), i guzy. Przy nowotworach kory nadnerczy (gruczolakach) u kobiet, ze względu na wzmożone tworzenie się androgenów (substancji działających jak męski hormon płciowy), obserwuje się zmiany w wyglądzie, pojawiają się cechy męskie (wąsik, broda, owłosienie ciała, rozwój mięśni i szkielet według typu męskiego). Niekiedy dołączają do tego objawy nek-ry charakterystyczne dla choroby Itsenko — Cushing. W przypadku guzów rdzenia nadnerczy, ze względu na zwiększone uwalnianie jego hormonu - adrenaliny, u pacjentów z napadowym wzrostem ciśnienia krwi obserwuje się wzrost poziomu cukru we krwi, wahania temperatury. Przy niewydolności funkcji warstwy korowej nadnerczy rozwija się szereg stanów patologicznych. stany związane głównie z obniżoną zdolnością adaptacyjną (adaptacją) do działania różnych szkodliwych czynników środowiska zewnętrznego i wewnętrznego (przeziębienie, głód, urazy fizyczne i psychiczne itp.), a także zaburzeniami gospodarki wodno-solnej.

Gdy aparat wyspowy trzustki jest uszkodzony, cukrzyca, os. Objawy to-rogo to wzrost zawartości cukru we krwi i jego przydział z moczem. Wynika to z niewystarczającej produkcji insuliny. Jeśli towarzyszy temu niedobór tworzenia innego hormonu trzustkowego - lipokainy, rozwija się stłuszczenie wątroby. W ciężkich postaciach cukrzycy następuje rozwój ketoza- zatrucie organizmu nadmiernie uformowanymi produktami metabolizmu tłuszczów. W przypadku guzów tkanki wyspowej ostry hipoglikemia(spadek poziomu cukru we krwi).

Opóźnienie lub przedwczesny i nadmierny rozwój pierwotnych i wtórnych cech płciowych łączy hl. przyb. z niedoczynnością lub nadczynnością gonad i wpływem ich hormonów. Niewydolność w rozwoju płci i niektórych innych gruczołów dokrewnych w okresie dojrzewania może być jedną z przyczyn infantylizmu,

Do leczenia chorób gruczołów V. s. obecnie szeroko stosowane różne leki hormonalne, energia promieniowania, operacyjne metody chirurgiczne, dietetyczne. odżywianie itp. Leczenie jest tym skuteczniejsze, im wcześniej wykryta zostanie choroba i postawiona zostanie prawidłowa diagnoza. Dzieci wymagają w tym względzie szczególnej uwagi. Dlatego przy najmniejszym podejrzeniu naruszenia funkcji któregokolwiek z gruczołów V. s. (stopniowa i postępująca utrata masy ciała lub otyłość, niewyjaśniony letarg lub nadmierna pobudliwość psychiczna i fizyczna, opóźniony lub przedwczesny wzrost wzrostu, obniżone zdolności umysłowe itp.), konieczne jest skierowanie dziecka do lekarza specjalisty.

Hormony ludzkie i ich funkcje: lista hormonów w tabelach i ich wpływ na organizm człowieka

Lit.: Sokolov D.D., Choroby endokrynologiczne u dzieci i młodzieży. M., 1952; Baranov VG, Choroby układu hormonalnego i metabolizmu, L., 1955; Vasyukova E. A. (red.), Przewodnik po endokrynologii klinicznej, M., 1958.

GL Shreiberg. Moskwa.

Źródła:

  1. Encyklopedia pedagogiczna. Tom 1. Rozdz. redaktor - A.I. Kairow i F.N. Pietrow. M., „Soviet Encyclopedia”, 1964. 832 kolumna. z ilustracjami, 7 arkuszy. chory.

Gruczoły dokrewne i ich znaczenie.

Wszystkie procesy zachodzące w naszym ciele są regulowane przez układ nerwowy i humoralny. Odgrywa znaczącą rolę w regulacji funkcji fizjologicznych organizmu układ hormonalny, wykonując swoje działania za pomocą chemikaliów przez płynne media organizmu (krew, limfa, płyn międzykomórkowy).

Układ hormonalny - tabela hormonów i ich funkcji

Głównymi narządami są układy – przysadka, tarczyca, nadnercza, trzustka, gonady.

Istnieją dwa rodzaje żołądź. Niektóre z nich posiadają kanaliki, którymi substancje są uwalniane do jamy ciała, narządów lub na powierzchnię skóry.

Nazywają się gruczoły wydzielania zewnętrznego. Zewnętrzne gruczoły wydzielnicze to gruczoły łzowe, potowe, ślinowe, żołądkowe, gruczoły, które nie mają specjalnych przewodów i wydzielają substancje do przepływającej przez nie krwi, nazywane są gruczołami dokrewnymi. Należą do nich przysadka mózgowa, tarczyca, grasica, nadnercza i inne.

Hormony- substancje biologicznie czynne. Hormony są produkowane w niewielkich ilościach, ale pozostają aktywne przez długi czas i są przenoszone przez organizm wraz z krwią.

Gruczoły dokrewne:

Przysadka. Znajduje się u podstawy mózgu. Hormon wzrostu. Świetnie wpływa na wzrost młodego organizmu.
nadnercza. Sparowane gruczoły przylegające do wierzchołka każdej nerki. Hormony – noradrenalina, adrenalina. Reguluje metabolizm wody, soli, węglowodanów i białek. Hormon stresu, kontrola aktywności mięśni, układ krążenia.
Tarczyca. Znajduje się na szyi przed tchawicą i na bocznych ścianach krtani. Hormonem jest tyroksyna. regulacja metabolizmu.
Trzustka. Znajduje się pod brzuchem. Hormonem jest insulina. Odgrywa ważną rolę w metabolizmie węglowodanów.
gonady. Męskie jądra to sparowane narządy zlokalizowane w mosznie. Kobieta - jajniki - w jamie brzusznej. Hormony - testosteron, hormony żeńskie. Uczestniczy w tworzeniu drugorzędowych cech płciowych, w rozmnażaniu organizmów.
Przy braku hormonu wzrostu wytwarzanego przez przysadkę występuje karłowatość, z nadczynnością - gigantyzmem. W przypadku niedoczynności tarczycy u dorosłych dochodzi do obrzęku - metabolizm jest zmniejszony, temperatura ciała spada, rytm skurczów serca jest osłabiony, a pobudliwość układu nerwowego zmniejsza się. W dzieciństwie obserwuje się kretynizm (jedna z form karłowatości), rozwój fizyczny, umysłowy i seksualny jest opóźniony. Brak insuliny prowadzi do cukrzycy. Przy nadmiarze insuliny poziom glukozy we krwi gwałtownie spada, towarzyszą temu zawroty głowy, osłabienie, głód, utrata przytomności i drgawki.

FUNKCJE GRUCZOŁÓW

Aktywność gruczołów dokrewnych jest kontrolowana przez liczne bezpośrednie i zwrotne połączenia w ciele. Głównym regulatorem ich funkcji jest podwzgórze, które jest bezpośrednio połączone z głównym gruczołem dokrewnym - przysadką mózgową, którego wpływ rozciąga się na inne gruczoły obwodowe.

FUNKCJE HIPOFIZY

Przysadka składa się z trzech płatów:

1) płat przedni lub przysadka gruczołowa,

2) udział pośredni i

3) płat tylny lub neuroprzysadka.

W adenopofizie główną funkcję wydzielniczą pełni 5 grup komórek, które wytwarzają 5 określonych hormonów. Wśród nich są hormony tropowe (łac. tropos - kierunek), które regulują funkcje gruczołów obwodowych oraz hormony efektorowe, które bezpośrednio oddziałują na komórki docelowe. Do hormonów tropikalnych należą: kortykotropina lub hormon adrenokortykotropowy (ACLT), który reguluje funkcje kory nadnerczy; hormon stymulujący tarczycę (TSH), który aktywuje tarczycę; hormon gonadotropowy (GTG), który wpływa na funkcje gruczołów płciowych.

Hormony efektorowe to somatotropina i hormon (GH) lub somatotropina, która warunkuje wzrost organizmu oraz prolaktyna, która kontroluje aktywność gruczołów sutkowych.

Uwalnianie hormonów przedniego płata przysadki jest regulowane przez substancje utworzone przez komórki neurosekrecyjne podwzgórza - neuropeptydy podwzgórza: stymulujące wydzielanie - liberyny i hamujące - z t i t oraz n i mi. Te substancje regulujące dostarczane są krwią z podwzgórza do przedniego płata przysadki, gdzie wpływają na wydzielanie hormonów przez komórki przysadki.

Somatotropina jest białkiem specyficznym dla gatunku, które warunkuje wzrost ciała (głównie zwiększa wzrost długości kości).

Prace nad inżynierią genetyczną polegające na wprowadzeniu szczurzej somatotropiny do aparatu genetycznego myszy umożliwiły uzyskanie dwukrotnie większych supermyszy. Jednak współczesne badania wykazały, że somatotropina organizmów jednego gatunku może zwiększać wzrost ciała u gatunków na niższych etapach rozwoju ewolucyjnego, ale nie jest skuteczna w przypadku bardziej rozwiniętych organizmów. Obecnie odkryto substancję mediatorową, która przenosi działanie hormonu wzrostu na komórki docelowe - somatomedynę, która jest produkowana przez komórki wątroby i tkanki kostnej. Somatotropina zapewnia syntezę białek w komórkach, akumulację RNA, usprawnia transport aminokwasów z krwi do komórek, wspomaga wchłanianie azotu, tworząc dodatni bilans azotowy w organizmie i pomaga w utylizacji tłuszczów. Wydzielanie hormonu somatotropowego wzrasta podczas snu, wysiłku fizycznego, urazów i niektórych infekcji.W przysadce mózgowej osoby dorosłej jego zawartość wynosi około 4-15 mg, u kobiet jej średnia ilość jest nieco wyższa. Szczególnie zwiększa stężenie hormonu wzrostu we krwi nastolatków w okresie dojrzewania. Podczas głodu jego stężenie wzrasta 10-15 razy.

Nadmierne uwalnianie somatotropiny we wczesnym wieku prowadzi do gwałtownego wzrostu długości ciała (do 240-250 cm) - gigantyzmu, a jego niedoboru - do opóźnienia wzrostu - karłowatości. Olbrzymy przysadkowe i karły mają proporcjonalną budowę, ale mają zmiany w niektórych funkcjach organizmu, w szczególności zmniejszenie funkcji wewnątrzwydzielniczych gonad. Nadmiar somatotropiny w stanie dorosłym (po zakończeniu wzrostu ciała) prowadzi do rozrostu jeszcze nie całkowicie skostniałych części kośćca - wydłużenia palców rąk i nóg, dłoni i stóp, brzydkiego wzrostu nosa, podbródka , a także do wzrostu narządów wewnętrznych. Ten stan nazywa się akromegalią.

Prolaktyna reguluje wzrost gruczołów sutkowych, syntezę i wydzielanie mleka (wydzielanie mleka zapewnia inny hormon - oksytocyna), pobudza instynkt macierzyński, a także wpływa na metabolizm wodno-solny w organizmie, erytropoezę, powoduje poród otyłość itp.

efekty. Jego uwolnienie jest odruchowo aktywowane przez akt ssania. Ze względu na to, że prolaktyna wspomaga istnienie ciałka żółtego i produkcję przez nie hormonu progesteronu, nazywana jest również hormonem luteotropowym.

Kortykotropina (hormon adrenokortykotropowy – ACTH) jest dużym białkiem, podczas tworzenia którego jako produkty uboczne uwalniana jest melanotropina (wpływająca na powstawanie barwnika melaniny) oraz ważny peptyd – endorfina, która zapewnia działanie przeciwbólowe w organizmie. Główny wpływ kortykotropiny na funkcje kory nadnerczy,

zwłaszcza na tworzenie glukokortykoidów. Dodatkowo powoduje rozpad tłuszczów w tkance tłuszczowej, zwiększa wydzielanie insuliny i somatotropiny. Stymuluj uwalnianie kortykotropiny różne stresujące bodźce - silny ból, przeziębienie, znaczny wysiłek fizyczny, stres psycho-emocjonalny. Przyczyniając się do wzmocnienia metabolizmu białek, tłuszczów i węglowodanów w sytuacjach stresowych, zapewnia wzrost odporności organizmu na działanie niekorzystnych czynników środowiskowych.

Lista hormonów

czyli jest to hormon adaptacyjny.

Tyreotropina (hormon stymulujący tarczycę - TSH) zwiększa masę tarczycy, liczbę aktywnych komórek, sprzyja wychwytywaniu jodu, co generalnie wzmaga wydzielanie jego hormonów. W rezultacie wzrasta intensywność wszystkich rodzajów metabolizmu, wzrasta temperatura ciała. Tworzenie TSH wzrasta wraz ze spadkiem temperatury zewnętrznej otoczenia i jest hamowane przez urazy, ból. Wydzielanie TSH może być spowodowane odruchem warunkowym – zgodnie z sygnałami poprzedzającymi ochłodzenie, czyli kontrolowanym przez korę mózgową. Ma to ogromne znaczenie przy procesach hartowania, treningu do niskich temperatur.

Hormony gonadotropowe (GTG) - follitropina i lutropina (nazywane również hormonami folikulotropowymi i luteinizującymi) - są syntetyzowane i wydzielane przez te same komórki przysadki, są takie same u mężczyzn i kobiet i działają synergistycznie. Te cząsteczki są chemicznie chronione przed zniszczeniem w wątrobie. HTG stymuluje tworzenie i wydzielanie hormonów płciowych, a także funkcję jajników i jąder. Zawartość HTG we krwi zależy od stężenia męskich i żeńskich hormonów płciowych we krwi, od odruchów podczas stosunku, od różnych czynników środowiskowych oraz od poziomu zaburzeń neuropsychiatrycznych.

Tylna przysadka wydziela hormony wazopresynę i oksytocynę, które powstają w komórkach podwzgórza, a następnie przez włókna nerwowe wchodzą do przysadki nerwowej, gdzie gromadzą się, a następnie są uwalniane do krwi.

Wazopresyna (łac.vas - naczynie, ciśnienie ciśnieniowe) ma podwójny fizjologiczny wpływ na organizm.

Po pierwsze powoduje zwężenie naczyń krwionośnych i wzrost ciśnienia krwi.

Po drugie, hormon ten zwiększa wchłanianie zwrotne wody w kanalikach nerkowych, co powoduje wzrost stężenia i zmniejszenie objętości moczu, czyli działa jako hormon antydiuretyczny (ADH). Jej wydzielanie do krwi jest stymulowane zmianami w metabolizmie wody i soli, aktywnością fizyczną i stresem emocjonalnym. Przygnębiony po spożyciu alkoholu

dochodzi do sekrecji wazopresyny (ADH), zwiększonego wydalania moczu i odwodnienia. W przypadku gwałtownego spadku produkcji tego hormonu dochodzi do moczówki prostej, objawiającej się patologiczną utratą wody przez organizm.

Oksytocyna stymuluje skurcze macicy podczas porodu, wydzielanie mleka przez gruczoły sutkowe. Jej wydzielanie jest wzmacniane impulsami z mechanoreceptorów macicy podczas jej rozciągania, a także wpływem żeńskiego hormonu płciowego – estrogenu.

Płat pośredni przysadki mózgowej prawie nie jest rozwinięty u ludzi, istnieje tylko niewielka grupa komórek, które wydzielają hormon melanotropowy, który powoduje powstawanie melaniny, pigmentu skóry i włosów. Zasadniczo tę funkcję u ludzi pełni kortykotropina przedniego płata przysadki mózgowej.

Poprzedni60616263646566676869707172737475Następny

ZOBACZ WIĘCEJ:

Funkcje układu hormonalnego

konserwacja homeostaza w ciele wymaga koordynacji wielu różnych układów i narządów.

Jeden z mechanizmy komunikacji między sąsiadującymi komórkami, jak również między komórkami i tkankami w odległych częściach ciała zachodzi oddziaływanie poprzez uwalnianie substancji chemicznych o nazwie hormony które są produkowane układ hormonalny.

Hormony są uwalniane do płynów ustrojowych, zwykle krwi.

1.5.2.9. Układ hormonalny

Krew przenosi je do komórek docelowych, gdzie hormony wywołują niezbędną reakcję.

Komórki wydzielające hormony często znajdują się w określonych narządach zwanych gruczoły dokrewne.

Komórki, tkanki i narządy wydzielające hormony są układ hormonalny.

Niektóre funkcje regulacyjne układ hormonalny obejmuje:

  • kontrola tętno,
  • kontrola ciśnienie krwi,
  • kontrola odpowiedź immunologiczna na infekcję
  • kontrola procesu hodowla, wzrost oraz rozwój organizm,
  • kontrola poziomu stan emocjonalny.

Gruczoły układu hormonalnego

Układ hormonalny składa się z:

Wiele innych narządów, takich jak wątroba, Skórzany, nerki i części trawienny oraz układy krążenia, wytwarzają hormony oprócz ich głównych specyficznych funkcji fizjologicznych.

Gruczoły dokrewne (gruczoły dokrewne) to gruczoły, które uwalniają hormony bezpośrednio do krwiobiegu przez przechodzące przez nie naczynia krwionośne, podczas gdy gruczoły zewnątrzwydzielnicze wydzielają swoje wydzieliny przez przewody lub rurki.

Przykładami gruczołów zewnątrzwydzielniczych są gruczoły potowe, ślinianki oraz gruczoły łzowe.

Rodzaje hormonów – hormony steroidowe i niesteroidowe oraz mechanizmy ich działania

Układ hormonalny wytwarza dwa główne typy hormonów:

  1. Hormony steroidowe
  2. Hormony niesteroidowe

Hormony steroidowe

Hormony steroidowe, takie jak kortyzol, są produkowane z cholesterol.

Każdy rodzaj hormonu steroidowego składa się z centralnej struktury czterech pierścieni węglowych z dołączonymi do nich różnymi łańcuchami bocznymi, które decydują o specyficznych i unikalnych właściwościach hormonu.

Wewnątrz komórek endokrynnych syntetyzowane są hormony steroidowe w gładka retikulum endoplazmatyczne.

Ponieważ hormony steroidowe są hydrofobowy, wiążą się z białkiem nośnikowym, które przenosi je przez krwioobieg.

Hormony steroidowe rozpuszczalne w tłuszczach mogą przenikać przez błonę komórkową docelową.

Wewnątrz komórki docelowej w cytoplazmie hormony steroidowe przyłączają się do cząsteczki białka receptorowego.

Ten kompleks hormon-receptor wchodzi następnie do jądra, gdzie wiąże się i aktywuje określony gen w cząsteczce. DNA.

Aktywowany gen wytwarza następnie enzym, który inicjuje pożądaną reakcję chemiczną w komórce.

Hormony niesteroidowe

Hormony niesteroidowe, takie jak adrenalina, składają się z białek, peptydów lub aminokwasów.

Te cząsteczki hormonów nie są rozpuszczalne w tłuszczach, więc zazwyczaj nie mogą dostać się do wnętrza komórki przez błonę plazmatyczną w celu wywołania swojego działania.

Zamiast tego wiążą się z receptorami na powierzchni komórek docelowych. To wiązanie z receptorami wyzwala następnie specyficzny łańcuch reakcji chemicznych w komórce.

gruczoł dokrewny Hormony Efekt hormonalny

Przysadka

Przysadka mózgowa (płat przedni (gruczołowa przysadka)) hormon wzrostu wspomaga wzrost tkanek ciała
Przysadka (przednia) prolaktyna promuje produkcję mleka
hormon stymulujący tarczycę stymuluje uwalnianie hormonów tarczycy
hormon adrenokortykotropowy stymuluje uwalnianie hormonów przez korę nadnerczy
hormon folikulotropowy stymuluje produkcję gamet
hormon luteinizujący stymuluje produkcję androgenów przez gonady u mężczyzn;
stymuluje owulację i produkcję estrogenu i progesteronu u kobiet
Przysadka mózgowa (tylny płat (neurohypofis)) hormon antydiuretyczny stymuluje reabsorpcję wody przez nerki
Przysadka (tylna) oksytocyna stymuluje skurcze macicy podczas porodu

Tarczyca

Tarczyca tyroksyna, trijodotyronina stymuluje metabolizm
Tarczyca kalcytonina obniża poziom Ca 2+ we krwi

Gruczoł przytarczyczny

parathormon (hormon przytarczyc) zwiększa poziom Ca 2+ we krwi

nadnercza

Nadnerkowy(kora) aldosteron zwiększa poziom Na + we krwi
Nadnercza (kora) kortyzol,
kortykosteron,
kortyzon

Nadnerkowy(rdzeń)

Nadnercza (rdzeń)

epinefryna,
noradrenalina
stymuluje reakcję walki lub ucieczki

Trzustka

Trzustka insulina obniża poziom glukozy we krwi
Trzustka glukagon podnosi poziom glukozy we krwi

szyszynka

szyszynka

melatonina reguluje rytm dobowy organizmu

grasica

Grasica (grasica)

tymozyna stymuluje produkcję i dojrzewanie limfocytów

1961. Receptory hormonalne znajdują się w komórkach narządów docelowych.

1962. W spoczynku główną formą transportu hormonów we krwi do celów jest ich transfer w połączeniu z określonymi białkami osocza.

1963. Hormon adrenokortykotropowy reguluje tworzenie i wydalanie glikokortykoidów.

1964. Hormon wzrostu praktycznie nie ma specjalnego narządu docelowego.

1965. Progesteron jest syntetyzowany w jajniku.

1966 Oksytocyna jest wydzielana przez podwzgórze i magazynowana w przysadce mózgowej.

1967. Tyroksyna jest syntetyzowana w tarczycy.

1968. Insulina, glukokortykoidy wpływają głównie na metabolizm węglowodanów.

1969. Glukokortykoidy są głównie zaangażowane w adaptację organizmu do silnych czynników.

1970. Adrenalina wpływa głównie na energię skurczów mięśni.

1971. Hormon somatotropowy jest syntetyzowany w przednim płacie przysadki mózgowej.

1972. Hormon antydiuretyczny jest syntetyzowany w podwzgórzu, gromadzi się w tylnym przysadce mózgowej, skąd dostaje się do krwi.

1973. Hormon adrenokortykotropowy jest syntetyzowany w przednim płacie przysadki mózgowej.

1974. Retencja wody w organizmie związana jest z działaniem hormonu ADH (antydiuretyk).

1975. Gruczoły wydzielania wewnętrznego nazywane są takimi gruczołami, które nie mają przewodów wydalniczych i wydzielają swoje sekrety do krwi.

1976. Jajniki i łożysko są gruczołami dokrewnymi.

1977. Gruczoły Brunnera i Lieberküna nie należą do gruczołów dokrewnych.

1978. Produktem wydzielania gruczołów dokrewnych są hormony.

1979. Hormony mają właściwość specyficzności – działają tylko na cel.

1980. Wysoka aktywność biologiczna jest nieodłączna dla hormonów.

1981. Hormony mają mały rozmiar cząsteczkowy, co pozwala im działać wewnątrzkomórkowo.

1982. Hormony są szybko niszczone przez tkanki.

1983. Stosowanie hormonów zwierzęcych w leczeniu ludzi jest możliwe, ponieważ hormony nie są specyficzne dla gatunku.

1984. W przysadce mózgowej wytwarzany jest hormon somatotropowy.

1985. Hormon wzrostu wpływa na cały organizm.

Hormon wzrostu stymuluje syntezę białek.

1987. Pod wpływem hormonu wzrostu bilans azotowy staje się dodatni.

1988. Hormon somatotropowy wspomaga mobilizację tłuszczów z magazynu.

1989. Hormon wzrostu wspomaga rozkład glikogenu.

1990. Hormon wzrostu przyczynia się do retencji wapnia, sodu i fosforu w organizmie.

1991. Hormon wzrostu przyspiesza wzrost ciała.

1992. Karłowatość przysadkowa to spowolnienie wzrostu ciała z brakiem hormonu somatotropowego.

1993. Gigantyzm to wzrost wzrostu i masy ciała pod wpływem nadmiaru hormonu somatotropowego.

1994. Przy nadmiarze hormonu somatotropowego u osoby dorosłej występuje akromegalia.

1995. Akromegalia to wzrost stóp, rąk, nosa, uszu, narządów wewnętrznych u osoby dorosłej z nadmiarem hormonu somatotropowego.

1996. W przysadce mózgowej wytwarzany jest hormon stymulujący tarczycę.

1997. Hormon stymulujący tarczycę wpływa na tarczycę.

Hormony i ich wpływ na stół ciała

Przy braku hormonu tyreotropowego dochodzi do niewydolności tarczycy.

1999. W przysadce mózgowej wytwarzany jest hormon adrenokortykotropowy.

2000. Hormon adrenokortykotropowy (ACTH) działa na nadnercza.

2001. Przy braku ACTH dochodzi do niewydolności nadnerczy.

2002. Przy nadmiarze ACTH dochodzi do nadczynności nadnerczy.

2003. Hormony gonadotropowe obejmują stymulację pęcherzyków i luteinizację.

2004. Intermedin jest produkowany w środkowym płacie przysadki mózgowej.

2005. Intermedin wpływa na kolor skóry.

2006. Produkcję intermedyny a promuje światło słoneczne.

2007. Przy braku intermedyny dochodzi do naruszenia pigmentacji skóry.

2008. Hormony nie są wytwarzane w przysadce mózgowej.

2009. Oksytocyna jest wytwarzana w podwzgórzu.

2010. Oksytocyna wpływa na macicę i gruczoły sutkowe.

2011. Oksytocyna indukuje skurcze macicy.

2012. Oksytocyna indukuje wyrzut mleka.

2013. W podwzgórzu powstaje hormon antydiuretyczny (ADH).

2014. ADH promuje reabsorpcję wody w przewodach zbiorczych.

2015. Niedobór ADH prowadzi do moczówki prostej.

2016. ADH podnosi ciśnienie krwi.

2017. Podwzgórze reguluje produkcję hormonów przysadki mózgowej.

2018. W podwzgórzu powstają czynniki uwalniające.

2019. Czynniki uwalniające sprzyjają syntezie hormonów przysadki mózgowej.

2020. W podwzgórzu nie ma czynników uwalniających prolaktynę.

2021. W podwzgórzu powstają czynniki hamujące (statyny).

2022. Kortykostatyna hamuje syntezę ACTH.

2023. Tyrostatyna hamuje syntezę hormonu stymulującego tarczycę.

2024. Somatostatyna hamuje syntezę hormonu wzrostu.

2025. Prolaktostatyna hamuje syntezę prolaktyny.

2026. Melatonina jest produkowana w szyszynce.

2027. Melatonina wspomaga rozjaśnianie skóry.

2028. Światło słoneczne zakłóca syntezę melatoniny.

2029. Melatonina spowalnia dojrzewanie.

2030. Hormon tyreotropowy nie jest wytwarzany w tarczycy.

2031. Jod jest niezbędny do syntezy hormonów tarczycy.

2032. Tyroksyna wpływa na wszystkie tkanki ciała.

2033. Tyroksyna sprzyja rozpadowi białek.

2034. Tyroksyna wspomaga rozkład tłuszczów.

2035. Tyroksyna wspomaga rozkład glikogenu.

2036. Tyroksyna zwiększa podstawowy metabolizm.

2037. Przy braku tyroksyny u dziecka rozwija się kretynizm.

2038. Przy braku tyroksyny u dorosłych pojawia się obrzęk śluzowaty.

2039. Przy nadmiarze tyroksyny pojawia się choroba Gravesa-Basedowa.

2040. Tyrokalcytonina jest produkowana w tarczycy.

2041. Tyrokalcytonina wpływa na kości.

2042. Tyrokalcytonina wpływa na wymianę wapnia i fosforu.

2043. Tyrokalcytonina wspomaga odkładanie wapnia w kościach.

2044. Antagonistą tyrokalcytoniny jest parathormon.

2045. Parathormon jest produkowany w przytarczycach.

2046. Parathormon wpływa na nerki, przewód pokarmowy i kości.

2047. Parathormon wypłukuje wapń z kości.

2048. Parathormon zwiększa wchłanianie zwrotne wapnia w kanalikach.

2049. Parathormon zwiększa wchłanianie wapnia w jelicie.

2050. Pod wpływem parathormonu wzrasta zawartość wapnia we krwi.

2051. Przy nadmiarze parathormonu dochodzi do osteoporozy.

2052. Przy braku parathormonu występują drgawki.

2053. Komórki alfa wysepek Langerhansa produkują glukagon.

2054. Komórki beta wysepek Langerhansa produkują insulinę.

2055. Insulina zwiększa przepuszczalność błony komórkowej dla glukozy.

2056. Pod wpływem insuliny spada zawartość glukozy we krwi.

2057. Insulina wspomaga syntezę tłuszczu z glukozy.

2058. Insulina wspomaga syntezę białek iaminokwasów.

2059. Przy niedoborze insuliny pojawia się cukrzyca.

2060. Zwiększa się ilość moczu u pacjenta z cukrzycą.

2061. Wraz ze wzrostem ilości insuliny w moczu pojawia się nadmiar glukozy i zgodnie z prawami osmozy niesie ze sobą wodę.

2062. Glukagon na metabolizm węglowodanów wspomaga rozkład glikogenu w wątrobie.

2063. Pod wpływem glukagonu wzrasta zawartość glukozy we krwi.

2064. Adrenalina i noradrenalina są syntetyzowane w rdzeniu nadnerczy.

2065. Adrenalina przyspiesza i nasila skurcze serca.

2066. Adrenalina obkurcza naczynia narządów wewnętrznych i rozszerza naczynia wieńcowe i mózgowe.

2067. Adrenalina rozluźnia mięśnie oskrzeli.

2068. Adrenalina obniża wydzielanie wszystkich soków trawiennych.

2069. Adrenalina działa hamująco na mięśnie gładkie przewodu pokarmowego.

2070. Adrenalina zwiększa podstawowy metabolizm.

2071. Adrenalina zwiększa produkcję ciepła i zmniejsza przenoszenie ciepła.

2072. Niewydolność nadnerczy nie prowadzi do żadnej choroby.

2073. Mineralokortykoidy są wytwarzane w kłębuszkowej strefie kory nadnerczy.

2074. Glukokortykoidy są wytwarzane w strefie pęczkowej kory nadnerczy.

2075. Androgeny i estrogeny są wytwarzane w siateczkowatej strefie kory nadnerczy.

2076. Mineralokortykoidy sprzyjają retencji sodu w organizmie.

2077. Mineralokortykoidy zwiększają wydalanie potasu z moczem.

2078. Mineralokortykoidy podwyższają ciśnienie krwi.

2079. Przy nadmiarze mineralokortykoidów dochodzi do nadciśnienia i obrzęku.

2080. Glikokortykosteroidy regulują metabolizm białek, tłuszczów i węglowodanów.

2081. Stres prowadzi do zwiększenia syntezy glukokortykoidów.

2082. Wraz z niedoborem glukokortykoidów następuje spadek odporności na szkodliwe działanie.

2083. Ciężka aktywność fizyczna zwiększa zawartość glikokortykoidów we krwi.

2084. Ból zwiększa zawartość glikokortykoidów we krwi.

2085. Androgeny są syntetyzowane w gonadach i korze nadnerczy.

2086. Estrogeny są syntetyzowane w gruczołach płciowych i korze nadnerczy.

2087. U kobiet zwiększona zawartość androgenów prowadzi do pojawienia się drugorzędowych męskich cech płciowych.

2088. U mężczyzn zwiększona zawartość estrogenu prowadzi do zaniku drugorzędowych męskich cech płciowych.

2089. Hormony tkankowe to hormony wytwarzane przez wyspecjalizowane komórki organizmu, które nie są związane z gruczołami dokrewnymi.

2090. Hormony tkankowe nie są syntetyzowane w skórze.

2091. Tymozyna jest syntetyzowana w grasicy.

2092. Tymozyna zwiększa liczbę limfocytów we krwi.

2093. Hormony, w porównaniu z nerwową regulacją funkcji, wolniej i nieekonomicznie realizują swoje działanie.

2094. Układ nerwowy kontroluje gruczoły dokrewne poprzez autonomiczny układ nerwowy, poprzez neurosekrecje i zmiany wrażliwości tkanek.

2095. Neurosekrecja to wydzielanie neurohormonu przez wyspecjalizowane komórki nerwowe do krwi (limfy).

2096. Pod wpływem metabolicznym hormonów rozumiemy wpływ na efektor, który zmienia metabolizm.

2097. Pod wpływem morfogenetycznym hormonów rozumiemy wpływ na procesy wzrostu i różnicowania komórek.

2098. Zasada sprzężenia zwrotnego jest nieodłączna w mechanizmie hormonalnej regulacji funkcji fizjologicznych.

2099. Hormonalna regulacja funkcji fizjologicznych odbywa się zgodnie z zasadą ujemnego sprzężenia zwrotnego.

2100. Podczas ćwiczeń wzrasta poziom insuliny we krwi. W tych warunkach wzrasta aktywność środkowego płata przysadki mózgowej.

2101. Po usunięciu przysadki u szczeniąt następuje zatrzymanie wzrostu fizycznego, rozwoju płciowego i umysłowego, niedorozwój gruczołów dokrewnych, ponieważ przysadka produkuje hormon somatotropowy, który stymuluje syntezę białek i wzrost.

2102. Tylny płat przysadki jest bogato zaopatrzony we włókna nerwowe pochodzące z jądra nadwzrokowego i przykomorowego podwzgórza.

2103. Pod wpływem stresu wzrasta poziom katecholamin we krwi, ponieważ zwiększa to ton współczulnego podziału autonomicznego układu nerwowego.

2104. Po przeszczepieniu narządu obowiązkowa jest terapia hormonalna kortykosteroidami, ponieważ kortykoidy tłumią reakcje immunologiczne odrzucenia przeszczepionego narządu.

2105. Insulina jest ważnym hormonem, ponieważ jest jedynym hormonem, który zwiększa przepuszczalność błon komórkowych na glukozę.

2106. Podwzgórze nazywa się dyrygentem orkiestry dokrewnej, ponieważ wszystkie gruczoły dokrewne są narządami docelowymi hormonów przysadki.

2107. Przy niewydolności funkcji hormonalnej trzustki wzrasta poziom glukozy we krwi.

⇐ Poprzedni34353637383940414243Następny ⇒

Data publikacji: 2014-12-30; Przeczytaj: 396 | Naruszenie praw autorskich do strony

Studopedia.org - Studopedia.Org - 2014-2018 rok (0,006 s) ...

KATEGORIE

POPULARNE ARTYKUŁY

2022 „kingad.ru” - badanie ultrasonograficzne narządów ludzkich